SAT I: Sample Test 12 Form 12 Name

Form 12
Name
SAT I: Sample Test 12
Math, Critical Reading, and Writing
Time: The exam is 3 hours and 20 minutes long. There are nine sections in this exam.
You will have 25 minutes to work on six of the sections, 20 minutes to work on two
sections, and 10 minutes to work on one of the sections. Section 4 was not
released by the College Board and is not included in this test. Section 4 does not
count toward your score.
Scoring:
• For each multiple-choice question you answer correctly you will receive
one point (whether the question is easy or hard).
• For questions you omit, you will receive no points.
• For wrong answers to multiple-choice questions, you will lose one-quarter
of a point.
• For a wrong answer to a “grid-in” math question, you don’t lose any points.
• The essay is scored on a scale from 1 to 6 by two different readers. The
total essay score is the sum of the two reader’s scores.
Guessing: An educated guess may improve your score. That is, if you can eliminate one or
more choices as wrong, you increase your chances of choosing the correct
answer and earning one point. On the other hand, if you can’t eliminate any
choices, omit the question and move on.
Answers: You may write in this test, but mark all answers on your answer sheet to receive
credit. Make each mark a dark mark that completely fills the oval and is as dark as
all your other marks. If you erase, do so completely.
DO NOT OPEN THE TEST UNTIL YOU ARE TOLD TO DO SO!
-1-
-2-
ESSAY
ESSAY
Time ESSAY
— 25 minutes
Time — 25 minutes
Time — 25 minutes
Turn to page 2 of your answer sheet to write your
ESSAY.
Turn
to page 2 of your answer sheet to write
The essay gives you an opportunity to show how effectively you
develop
ideas. You
should,
take
Thecan
essay
gives and
you express
an opportunity
to show
howtherefore,
effectively
you can develop and exp
Turn
to page
of ideas
yourlogically
answer
to
write
your
ESSAY.
care to develop your point
of view,
present2your
and
clearly,
use
language
precisely.
care
tosheet
developand
your
point
of
view,
present your ideas logically and clearly, and use l
Yourgives
essay you
mustan
beopportunity
written on thetolines
provided
on your answer
receive
nothe
other
paper
onshould,
which
to
write.
Yoursheet—you
essay
mustwill
beand
written
on
lines
provided
on your
answer take
sheet—you will rec
The essay
show
how effectively
you
can
develop
express
ideas.
You
therefore,
You
will
have
enough
space
if
you
write
on
every
line,
avoid
wide
margins,
and
keep
your
handwriting
to
a
reasonable
size.wide margins, and keep
You
will
have
enough
space
if
you
write
on
every
line,
avoid
care to develop your point of view, present your ideas logically and clearly, and use language precisely.
Remember that people who are not familiar with your handwriting
will read
youwho
write.
write or
print
so handwriting
that what will read what you
Remember
thatwhat
people
are Try
not to
familiar
with
your
you are writing is legible to those readers.
you are writing is legible to those readers.
Your essay must be written on the lines provided on your answer sheet—you will receive no other paper on which to write.
You will
space
if youtowrite
everyonline,
widehave
margins,
andNOT
keep
your
handwriting
reasonable
size.below. DO NO
Youhave
haveenough
twenty-five
minutes
write on
an essay
the avoid
topic assigned
below.
DO
WRITE
ON ANOTHER
You
twenty-five
minutes
to write
an essay to
onaTOPIC.
the
topic assigned
Remember
that peopleESSAY
who are
not familiar
your handwriting
will
read
what
you
write.
Try
to
write
or
print
so
that
AN OFF-TOPIC
WILL
RECEIVEwith
A SCORE
OF ZERO.
AN OFF-TOPIC ESSAY WILL RECEIVE A SCORE OF ZERO. what
you are writing is legible to those readers.
Think carefully about the issue presented in the following excerpt
the assignment
Thinkand
carefully
about thebelow.
issue presented in the following excerpt and the assignment
You have twenty-five minutes to write an essay on the topic assigned below. DO NOT WRITE ON ANOTHER TOPIC.
AN OFF-TOPIC ESSAY
WILLweRECEIVE
OFstart
ZERO.
The people
call heroes A
doSCORE
not usually
out as unusual. The
Often
they are
ordinary
people
people
we call
heroes
do not usually start out as unusual. Often th
subject to ordinary human weaknesses—fear, doubt, and self-interest.
fact, they
live ordinary
subject toInordinary
human
weaknesses—fear, doubt, and self-interest.
Think carefully aboutlives
theuntil
issuethey
presented
in the
following
excerpttoand
below.
distinguish
themselves
by having
dealthe
withassignment
an injustice
or adistinguish
difficult situation.
lives
until they
themselves by having to deal with an injus
Only then, when they must respond in thought and in action toOnly
an extraordinary
challenge,
do
then, when they
must respond
in thought and in action to an ext
people begin to know their strengths and weaknesses.
people begin to know their strengths and weaknesses.
We often hear that we can learn much about someone or something just by casual observation.
We are not required to look beneath the surface or to question how something seems. In fact, we
are urged to trust our impressions, often our first
impressions, of how
a and
person
oranaessay
situation
Assignment: Do people learn who they are only when theyAssignment:
are forced into action?
Planlearn
writethey
in which
Do people
who
are only
when you
they are forced into action?
seemsdevelop
to be. your
Yet appearances
can
be
misleading.
What
“seems”
isn’t
always
what
is.
point of view on this issue. Support your position with reasoning and examples taken from your
develop your point of view on this issue. Support your position with
reading, studies, experience, or observations.
reading, studies, experience, or observations.
Assignment: Is the way something seems to be not always the same as it actually is? Plan and write an essay in which you
your point
of view
on this
issue.
Support
your
position
with
reasoning
and write
examples
taken
from your
DO NOT develop
WRITE YOUR
ESSAY
IN YOUR
TEST
BOOK.
You
will receive
credit
only
for what
on your
answer
DO NOT
WRITE
YOUR
ESSAY
INyou
YOUR TEST
BOOK.
You will receive credit o
reading,
studies,
experience,
or
observations.
sheet.
sheet.
BEGIN WRITING YOUR ESSAY ON PAGE 2 OF THE ANSWER
WRITING
ESSAY
PAGE 2 OF THE A
DO NOT WRITE YOUR ESSAY
IN YOUR TEST BOOK. You will receive credit BEGIN
only forSHEET.
what youYOUR
write on
yourON
answer
sheet.
BEGIN WRITING YOUR ESSAY ON PAGE 2 OF THE ANSWER SHEET.
If you finish before time is called, you may check your work on this section only.
If you finish before time is called, you may check your wo
Do not turn to any other section in the test.
Do not turn to any other section in the t
If you finish before time is called, you may check your work on this section only.
Do not turn to any other section in the test.
-3-
line 28) 19. The23.
In lines
the author
refers
to the Goldilocks
fairy 23. In lines 53-59 the author refers to the Goldilocks fairy
author
uses 53-59
the phrase
“this new
intimacy”
(line 28)SECTION
2
(“Yet . . . water”) in order to make which point
to refer tale
to the
tale (“Yet . . . water”) in order to make which point
Time
—
25
minutes
about a particular planet?
about a particular planet?
e repre(A) hands-on quality of the learning experience repre- 24 Questions
(A) The
planet’s
environment
sented
by the
shoebox
diorama may be conducive
(A) The planet’s environment may be conducive
e about
to a result
some scientistsnow
are have
eagerabout
to find.
(B) understanding
that
nonspecialists
to a result
scientistsinarethis
eager
to find.
Turn to Section 2 (page 4) of your answer sheet to answer
the some
questions
section.
(B) The planet’s
atmosphere was once thought
meteorological
phenomena
(B) The planet’s atmosphere was once thought
gical
to be too cold
to theory
supportthat
biological
life.
(C) general acceptance
of the
biological
to be too cold to support biological life.
Directions:
For
each
question
in this
section, select
the choices
given
and fill in used
the corresponding
(C)once
The
simple
methods
astronomers
usedthe
to best answer from
life
existed
on
Mars
(C) among
The simple
methods
astronomers
to
circle
the answer
sheet.the that
bout
discover
features
of thishave
planet
resem(D) on
increased
knowledge
scientists
about
discover the features of this planet resemthe explorations of curious children.
our solarble
system
ble the explorations of curious children.
on
(D)
speculations
about
(E) way
thatScientists’
events onwishful
one planet
affect those
on the
Scientists’
wishful speculations
about theour con4.(D)
Many
of our memories
are -------, escaping
Each sentence
below
has one
or two
blanks,
eachlittle
blank
existence
of this
planet
deserve
more
another
existence
deserve
sciousness
justofasthis
we planet
strain to
recall little
a facemore
or a name.
indicating that something
has been
omitted.
credence than
a fairy
tale. Beneath
credence than a fairy tale.
elusive
(B) trial
pervasive
(C)
fiveafter
words
or sets
ofand
words
A
(E)are
Only
trial
errorlabeled
did astronomers
(E)(A)Only
after much
and error
didunvaried
astronomers
20.theInsentence
line 33,
“crude”
mostmuch
nearly
means
(D)
insensitive
(E)
impractical
through E. Choose
the wordthe
orprecise
set of words
that,
determine
location
of when
this planet.
determine the precise location of this planet.
(A) natural
and unaltered
inserted
in the sentence,
best fits the meaning of the
(B) 24.
rough
and
inexpert
sentence
as
a
whole.
Although
Gordon
was
rigorously
objective
Which of the following, if true, would the “hopeful
24.5.Which
of theCaroline
following,
if true,
would
the “hopeful
(C) obvious
in
her
journalistic
writing,
her
lively
and
------private
scientists” (line 62) most likely interpret as evidence
scientists” (line 62) most likely interpret as evidence
Example:
(D) vulgar
correspondence
------a
delightful
capacity
for
biting
of the potential for life on Mars?
of the potential for life on Mars?
(E)
nonspecific
commentary on the social scene.
Hoping
to ------- the dispute, negotiators proposed
(A) Mars
by the
same planetary
(A) Mars was affected by the same planetary collision
a compromise
thatwas
theyaffected
felt would
be ------to both collision
(A) that
incisive
. . disguised
that caused
extinction
of dinosaurs.
t likely 21. In
caused
the extinction of dinosaurs.
lineand
42, management.
the author
refersthe
to Pluto’s
moon
most likely
labor
(B)
eloquent
. derided
(B)
Mars
had
a
very
mild
atmospheric
temperature
(B) Mars had a .very
mild atmospheric temperature
in order to
(C) millions
dispassionate
. . demonstrated
(A) enforce . millions
. useful of years ago.
of years
ago.
overed
(A)
illustrate
a feature
ourenvironment
solar systematdiscovered
exuberant
. . minimized
(B) end
. . divisive
(C)
Mars
had aof
wet
one time in the
(C)(D)Mars
had a wet
environment at one time in the
since thepast.
childhood
(E) past.
entertaining . . exhibited
(C) overcome
.author’s
. unattractive
en
(B)
an object
too that
small
scaleMars
to have
been rocks
(D) cite
extend
.The
. satisfactory
(D)
rock
fellinfrom
resembled
(D) The rock that fell from Mars resembled rocks
included
in theonauthor’s
diorama
6. An effective
of a debating
(E) resolve
. .found
acceptable
the Antarctic
ice field.
found onmember
the Antarctic
ice field.team must focus
oon
(C) draw
and our
own
on thethat
------and avoid
-------few
arguments.
(E)a parallel
The rockbetween
that fellitfrom
Mars
hadmoon
very few
(E)clearly
The rock
fell issue
from Mars
had very
children
(D) contrast the
scientific
curiosity
of
today’s
children
microscopic fissures.
(A) microscopic
equivocal . .fissures.
obstreperous
with------that of
years
ago from their
1. Despite
onchildren
taking rare
tamarins
(B)
designated
.
.
pertinent
ment to
(E)habitat,
emphasize
the
need
for
a
greater
commitment
to
the illegal trade in the tiny monkeys
(C)
comprehensive
. . general
space
exploration
remains
-------.
(D) principal . . peripheral
(A)reasoning
commendations
. . obligatory
(E) subtle . . significant
22. The
process presented
in lines 49-53
(B)
consultations
.
.
predominant
(“As . . . stars”) is best described as
(C) restrictions . . local
7. The ------- with which merchants and landowners in
(A)
on an untested theory
(D)inference
penaltiesbased
. . illicit
early-nineteenth-century Maryland and Virginia ------(B)(E)extrapolation
similar situations
prohibitionsfrom
. . active
Joshua Johnston’s professional services attests to his
vers
(C) analysis of a single case by multiple observers
artistic skill as a portrait painter.
n
hypothesis aconfirmed
by direct
observation
2.(D)
Representing
round world
on a flat
surface is impos(A) avidness . . sought
nce
(E)sible
comparison
of theory
without some
-------:with
thephysical
Mercatorevidence
projection
(B) diffidence . . purchased
map shows Greenland as over ten times larger than
(C) patience . . replaced
Mexico, a country in fact only slightly smaller than
(D) elegance . . regarded
Greenland.
(E) zealousness . . overlooked
(A) oversight
(B) simplification
(C) distortion
(D) sophistication
8. The man’s colleagues characterized him as ------(E) superficiality
because he had an irritable, quarrelsome disposition.
(A) tyrannical
(B) disingenuous
3. The highly publicized redesign of the car is essentially
(C)
sanctimonious
(D) cantankerous
-------: the exterior has been updated, but the engine
(E)
morose
remains unchanged.
led, you may check your
work
on this
section
only.
If you
finish
before
time
is called, you may check your work on this section only.
(A) intuitive
(B)
cosmetic
(C) incoherent
n to any other section in the test.
Do
not
(D) consequential
(E) retroactive turn to any other section in the test.
STOP
STOP
-4-
acy” (line 28) 19. The23.
In lines
the author
refers
to the Goldilocks
author
uses 53-59
the phrase
“this new
intimacy”
(line 28) fairy 23. In lines 53-59 the author refers to the Goldilocks fairy
The passages
are. .followed
basedwhich
on their
content; questions
following
a pairinoforder
related
passages
maypoint
also
(“Yet
. water”)by
in questions
order to make
point
to refer tale
tobelow
the
tale (“Yet
. . . water”)
to make
which
be based on the
relationship
between
the
paired
passages.
Answer
the
questions
on
the
basis
of
what
is
stated
or
implied
in
the
about a particular planet?
about a particular planet?
erience repre- passages
(A) and
hands-on
of the material
learning that
experience
in anyquality
introductory
may berepreprovided.
(A) The
planet’s
environment
sented
by the
shoebox
diorama may be conducive
(A) The planet’s environment may be conducive
w have about
to a result
some scientistsnow
are have
eagerabout
to find.
(B) understanding
that nonspecialists
to a result some scientists are eager to find.
Questions 9-12
on the
following
passages.
(B) are
Thebased
planet’s
atmosphere
was
once thought
meteorological
phenomena
(B) The planet’s atmosphere was once thought
10. The author
Passage
would most
likelylife.
respond
biological
to be too cold
to theory
supportthat
biological
life.
(C) general acceptance
of the
biological
to beoftoo
cold to2support
biological
to
statement
lines 4-5,
Passage 1 used
(“it rarely
Passage 1 islife
byonce
Dorothy
Sayers;
Passage
2 is adapted
(C)
Theexisted
simple
methods
astronomers
used to
on
Mars
(C)theThe
simpleinmethods
astronomers
to . . .
passion”),
by
from
a
work
by
Raymond
Chandler.
ave about
the that
features
of thishave
planet
resem(D) increaseddiscover
knowledge
scientists
about
discover the features of this planet resemthe explorations of curious children.
our solarble
system
ble thethat
explorations
of curious
(A) arguing
this approach
limits children.
the characters’
Passage
those on
(D)
speculations
about
(E) 1way
thatScientists’
events onwishful
one planet
affect those
on the
(D) Scientists’
wishful speculations about the
development
existence of this planet deserve little more
another
existence
this writers
planet deserve
little stories
more
(B) denying
thatofmost
of detective
The detective
story does not and cannot attain the
credence than a fairy tale.
credence
than
a
fairy
tale.
rely on formulas
loftiest level of literary achievement. Though it deals
(E)“crude”
Only after much
trial
and error did astronomers
(E) agreeing
Only after
much
trialemotions
and errorare
didout
astronomers
20. In
33,
nearly
means
(C)
that
strong
of place
with
theline
most
desperate most
effects
of rage,
jealousy, and
determine the precise location of this planet.
determine
the
precise
location
of
this
planet.
in
detective
stories
Line revenge,
rarely and
touches
the heights and depths of
(A) itnatural
unaltered
(D)
conceding
that
great
literature
is
seldom
5 human
It
presents
us with an accomplished
(B)passion.
rough
and
inexpert
24.
Which
of
the following, if true, would the “hopeful
24. Whichcommercially
of the following,
if true, would the “hopeful
successful
fact, (C)
and looks
upon death with a dispassionate eye. It
obvious
scientists” (line 62) most likely interpret as evidence
scientists”
(line
62)
most
likely
evidence
(E) concurring that readers
are interpret
primarilyasinterested
does (D)
not show
us the inner workings of the murderer’s
vulgar
of the potential for life on Mars?
of
the
potential
for
life
on
Mars?
in
plot
mind—
must not, for the identity of the criminal is
(E) it nonspecific
Mars
affected
the same
planetary collision
hidden until (A)
the end
of was
the book.
Thebymost
successful
(A) Mars was affected by the same planetary collision
11. Which that
of the
following
characteristics
of detective
thatcontrive
caused
extinction
of dinosaurs.
10 writers
are those
totokeep
themoon
story
running
n most likely
caused
the extinction
of dinosaurs.
21. In line
42, thewho
author
refersthe
Pluto’s
most
likely
stories
presented
in
Passage
1
would
be
(B)
Mars
had
a
very
mild
atmospheric
temperature
frominbeginning
to
end
upon
the
same
emotional
level,
(B) Mars had a very mild atmospheric LEAST
temperature
order to
likely to
be attributed
to ago.
the “pattern” mentioned
of years ago.
and it is better to errmillions
in the direction
of too little feeling
millions
of years
m discovered
illustrate
a feature
ourenvironment
solar systematdiscovered
in
19, Passage
2 ?environment at one time in the
(C) Mars
had aof
wet
one time in the
than (A)
too much.
(C)lineMars
had a wet
since thepast.
author’s childhood
past.
(A) “cannot attain the loftiest level of literary
ve been
Passage
(B) 2cite(D)
an object
too that
small
scaleMars
to have
been rocks
The rock
fellinfrom
resembled
(D) The
rock that fell
from1-2)
Mars resembled rocks
achievement”
(lines
includedfound
in theonauthor’s
diorama
the Antarctic
ice field.
foundwith
on the
Antarctic
ice field.
I
think
what
was
really
gnawing
at
Dorothy
Sayers
in
(B)
“deals
the
most
desperate
effects of
wn moon
(C) draw
a parallel
itfrom
and our
own
(E)the
The
rockbetween
that
fellwas
Mars
hadmoon
very
few
(E) The
rock
that
fell
from
Mars
had
very
few
15 her critique of
detective
story
the
realization
that
rage,
jealousy,
and
revenge”
(lines
2-4)
day’s children
(D) contrast the
scientific curiosity
of today’s children
microscopic
fissures.
microscopic
fissures.
her kind of detective
story was an
arid formula unable to
(C) “presents us with an accomplished fact”
with that of children years ago
its own implications. If the story started to be about
(lines 5-6)
mmitment to satisfy
(E) emphasize the need for a greater commitment to
real people, they soon had to do unreal things to conform
(D) “looks upon death with a dispassionate eye”
space exploration
to the artificial pattern required by the plot. When they did
(line 6)
20 unreal things, they ceased to be real themselves. Sayers’
(E)
“does
not show us the inner workings of the
49-53
22. The reasoning process presented in lines 49-53
own stories show that she was annoyed by this triteness.
murderer’s
mind” (lines 7-8)
(“As . . . stars”) is best described as
Yet she would not give her characters their heads and let
y
them(A)
make
their own
mystery.
inference
based
on an untested theory
12. Passage 1 suggests that Sayers would most likely
(B) extrapolation from similar situations
respond to lines 17-20, Passage 2 (“If the story started
observers
(C) analysis
of a single
by multiple
observers
9. Which
best describes
the case
relationship
between
. . . themselves”), by pointing out that
vation
the
passages?confirmed by direct observation
(D)two
hypothesis
evidence
(A) great writers seldom explore the range of human
(E) comparison of theory with physical evidence
(A) Passage 1 explains the evolution of a genre,
emotions
while Passage 2 challenges the notion of
(B) detective stories do not address the consequences
a distinct genre.
of people’s emotions
(B) Passage 1 discusses the constraints of a genre,
(C) detective stories are driven by the plot, not by
while Passage 2 contends that many of these
the characters
constraints are self-imposed.
(D) readers of detective stories prefer unrealistic
(C) Passage 1 celebrates a genre, while Passage 2
situations
points out its deficiencies.
(E) real people often act in ways that are unexpected
(D) Passage 1 explains the popularity of a genre,
while Passage 2 questions its commercial
success.
(E)
Passage
compares
genre
unfavorably
s called, you may check your
work
onathis
section
If1 you
finish
before
time only.
is called, you may check your work on this section only.
to
other
types
of
writing,
while
2 to any other section in the test.
t turn to any other section in the test.
DoPassage
not turn
argues that the genre has unique features.
STOP
STOP
-5-
line 28) 19.
In
lines
53-59
the on
author
refers
to thepassage.
Goldilocks
in the
city streets.
Butauthor
my adversaries
performed
similar
The23.
author
usesare
thebased
phrase
“this
intimacy”
(line 28) fairy 23.
In lines
53-59 the
refers to the
Goldilocks
fairy
Questions
13-24
thenew
following
(“Yet . . . water”) in order to make which point
experiments
different
they were
to refer tale
to the
tale (“Yet with
. . . water”)
in results.
order toPerhaps
make which
pointnot
about
a particular
careful
my planet?
procedures. The neck of the flask must
abouttoa follow
particular
passage
is adapted
fromplanet?
a series in which a college
e repre- This
(A) hands-on quality of the learning experience reprebe heated first to kill the bacteria on the glass; then a heated
professor dramatizes
the
lectures
of
famous
scientists
from
(A) The
planet’s
environment
sented
by the
shoebox
diorama may be conducive
(A) The planet’s environment may be conducive
50 instrument must be used to break the tip of the flask as it is
the past. Here hetospeaks
as some
Louisscientists
Pasteur (1822-1895).
e about
a result
are have
eagerabout
to find.
(B) understanding
that nonspecialists
now
to a result some scientists are eager to find.
held high above the head. Immediately thereafter the flask
In this part(B)
of the
lecture,
Pasteur
has just
described
his
The
planet’s
atmosphere
was
once thought
meteorological
phenomena
(B) The planet’s atmosphere was once thought
must be sealed again in a flame [Pasteur demonstrates the
discovery of the effect
microbes
that
gical
to be of
tooheating
cold
tocertain
supportthat
biological
life.
(C) general acceptance
of the
theory
biological
to be too cold to support biological life.
procedure]. In these difficult researches, while I sternly
infect bottled
beverages
(the
process
later
named
(C)once
Theexisted
simpleon
methods
life
Mars astronomers used to
(C) The simple methods astronomers used to
object to frivolous contradictions, I feel nothing but gratpasteurization). discover the features of this planet resembout
(D) increased knowledge that scientists have about
discover the features of this planet resem55 itude toward those who warn me if I should be in error.
the explorations
of curious
children.
our undesirable
solarble
system
ble the explorations of curious children.
But these
microbes! Where
and how
did
I then devised a conclusive experiment. I boiled a nutrion
(D)
Scientists’
speculations
about
(E)arise?
way
that
events onwishful
one
planet
affect
those
on the
(D) Scientists’ wishful speculations about the
they
By
spontaneous
generation,*
as some
believe?
tious infusion in a flask with a long curved neck like this
this planet
deserveand
little
another
existence of this planet deserve little more
When I began
to existence
ask these of
questions
of myself
of more
my
one. The tip of the neck was not sealed but left open to the
credencemy
than
a fairy
tale.said: “Oh, no,
credence than a fairy tale.
Line students and colleagues,
close
friends
outside air. Thus, there was no hindrance to the entrance of
(E)your
Only
after
trial
and error
did astronomers
(E) Only after much trial and error did astronomers
33,
“crude”
most
nearly
means
do In
notline
waste
time
on much
such
worthless
philosophical
5 20.
60 fresh air with its “vital force” as claimed by the advocates
the precise
location
of this planet.
determine the precise location of this planet.
problems. Many determine
a scientist has
floundered
and perished
in
of spontaneous generation. But bacteria in the entering air
(A) natural and unaltered
the quagmire of spontaneous generation.” I replied: “But
would be trapped by the walls of the long glass tube. The
(B) 24.
rough
and inexpert
Which
following,
if true,With
would
theexcep“hopeful
24. Which of the following, if true, would the “hopeful
the origin
of
life isofathe
profound
problem.”
few
fluid remained sterile so long as the flask was maintained
(C) obvious
62) most likely
interpret
as evidence
scientists” (line 62) most likely interpret as evidence
tions, past scientists”
discourses (line
on spontaneous
generation
have
been
in the vertical position. If, however, I contaminated the
(D) vulgar
of
the
potential
for
life
on
Mars?
of the potential for life on Mars?
10 metaphysical exercises conducted with great passion, but
65 broth by allowing some of it to flow into the neck and then
(E) nonspecific
without adding
to
our
scientific
knowledge.
(A) Mars was affected by the same planetary collision
back
the was
flask,affected
putrefaction
followed.
So we
(A)into
Mars
by thepromptly
same planetary
collision
I could not setthat
aside
my burning
desire to
a little
caused
extinction
ofbring
dinosaurs.
t likely 21. In
see that life
not the
ariseextinction
spontaneously.
Life comes only
thatdoes
caused
of dinosaurs.
line 42, the author
refersthe
to Pluto’s
moon
most likely
stone, God(B)
willing, to the frail edifice of our knowledge of
from
(B)life.
Mars had a very mild atmospheric temperature
in order to Mars had a very mild atmospheric temperature
the deep mysteries
of life of
andyears
death,
where all our intellects
millions
ago.
millions of years ago.
overed15 have
lamentably
failed.
In
defense
nonapplied
science
(A)so illustrate
a feature
our
solarof
system
(C) Mars
had aof
wet
environment
atdiscovered
one time
in the
living matter directly
lifeless
matter
*The
(C)supposed
Marsorigination
had a wetofenvironment
at onefrom
time
in the
I have repeatedly
told
my students
that without theory,
since thepast.
author’s
childhood
past.
en
practice
is (D)
but
Only
theory
is able
toresembled
cause
(B) cite
an routine.
object
too
small
scale
to have
beenthe rocks
The rock
that
fellinfrom
Mars
(D) The rock that fell from Mars resembled rocks
spirit of invention
to
and
develop.
It
is
important
that
includedfound
in arise
theonauthor’s
diorama
the Antarctic ice field.
foundofonthe
thelecture
Antarctic
field.
13.
The focus
is onice
how
Pasteur
oon
students
should
not share
the fell
opinion
those
who
disdain
(C) draw
itfrom
andof
our
own
moon
(E)a parallel
The
rockbetween
that
Mars
had
very
few
(E) The rock that fell from Mars had very few
children20 everything
in science
that has curiosity
no immediate
application.
(D) contrast
the
scientific
of today’s
childrenIn
(A) microscopic
disproved anfissures.
erroneous theory
microscopic
fissures.
science, chance
favors
only theyears
mindago
that is prepared.
with that
of children
(B) documented and published his experiments
ment to
I repeat:
in science,the
chance
favors
only the
mind that to
(E) emphasize
need for
a greater
commitment
(C) developed a process for killing microbes
space exploration
is prepared.
(D) applied his findings on spontaneous generation
I first confirmed the experiments of the Italian abbé,
to new problems
25 22.
Lazzaro
Spallanzani,
known
also forinhis
studies
in gastric
(E) contributed to the improvement of laboratory
The reasoning
process
presented
lines
49-53
digestion.
made aisnutritious
broth, as
put it in a flask such
(“As . . I. stars”)
best described
research standards
as this [Pasteur holds up a large flask containing a brown
(A) inference based on an untested theory
solution], heated it to violent boiling, and then sealed the
14. In the lecture, Pasteur concludes that the answer to the
(B) extrapolation from similar situations
neck
of
the
flask
in
a
flame.
My
results
agreed
with
those
question “Where and how did they arise?” (lines 1-2) is
vers
(C) analysis of a single case by multiple observers
30 of Spallanzani: the broth remained pure. But if the neck
n
(D) hypothesis confirmed by direct observation
(A) spontaneously
be broken to admit air, the broth soon became putrid. My
nce
(E) comparison of theory with physical evidence
(B) from airborne bacteria
critics said that the heating made the air in the flask unfit
(C) from impurities in the original broth
for spontaneous generation. Only when fresh air is admitted
(D) from the curved neck of a flask
can life begin anew. I argued in vain—even before our
(E) from a broken flask
35 Academy of Sciences—that the putrefaction was caused by
admission of bacteria. More convincing experiments were
needed.
I opened flasks of sterilized broth in the cellar of the
Paris observatory, where the air was still. Only one flask
40 out of ten became putrid, whereas eleven flasks out of
eleven opened in the courtyard quickly acquired a rich
growth of bacteria. I journeyed to Mt. Montanvert in the
Alps,check
where your
I opened
twenty
flaskssection
of sterilized broth. Only
led, you may
work
on this
If you
finish
before time only.
is called, you may check your work on this section only.
one
became
putrid.
I
concluded
that
the air in the cellar and
n to any other section in the test.
Do not turn to any other section in the test.
45 the air above the glacier were freer of bacteria than the air
STOP
STOP
-6-
macy” (line 28) 15.
In two
lines
53-59
the (lines
author
refers
to the Goldilocks
19.InThe
author
usesparagraphs
the phrase
“this new
intimacy”
(line
the23.
first
1-23),
Pasteur
is 28) fairy
(“Yet . .with
. water”) in order to make which point
to refer tale
toconcerned
the
primarily
about a particular planet?
erience repre(A) summarizing
hands-on quality
of the of
learning
experienceabout
repre(A)
the results
his experiments
(A) The
planet’s
environment
sented
by the
shoebox
diorama may be conducive
spontaneous
generation
w have about
to
a result
some
are
eagerout
to find.
(B) criticizing
understanding
that
have
about
(B)
those
whononspecialists
have scientists
taken thenow
passion
(B)
The planet’s
atmosphere was once thought
phenomena
ofmeteorological
science
t biological
to his
be too
cold
to theory
support
biological
life.
(C) establishing
general acceptance
of the
that
biological
(C)
motivation
for studying
the origin
(C)
Theexisted
simpleon
methods
once
Mars astronomers used to
oflife
microbes
have about
discover
the experiments
features
of thishave
planet
resem(D) attacking
increasedcritics
knowledge
that
scientists
about
(D)
of his
the
explorations
of curious
children.
our solarble
system
(E) correcting
the
impression
that he
is concerned
those on
(D)
Scientists’
speculations
about
(E) way
that
events
onwishful
one that
planet
affect
those
on the
only
with
experiments
have
immediate
another existence of this planet deserve little more
application
credence than a fairy tale.
(E)
Only after
much
trial
andprimarily
error didtoastronomers
20.The
In line
33,
“crude”
most
nearly
means
16.
word
“quagmire”
(line
7) is
used
emphasize the determine the precise location of this planet.
(A) natural and unaltered
(B) 24.
rough
inexpert
(A)
state
of and
scientific
ignorance in
the 1800’s
Which
of
the following,
if true,
would the “hopeful
(C) futility
obvious
(B)
of
a
particular
line
of
research
scientists” (line 62) most likely interpret as evidence
(D) moral
vulgar
(C)
faced
scientists
like Pasteur
of dilemma
the potential
forby
life
on Mars?
(E) failure
nonspecific
(D)
of some to distinguish between pure and
(A) Mars
was affected by the same planetary collision
applied
science
that
caused
extinction
dinosaurs.
n most likely 21.(E)
In line
42, theof
author
refersthe
to Pluto’s
moon
mostfor
likely
tendency
unsuccessful
scientists
tooflook
(B)
Mars
had
a
very
mild
atmospheric
temperature
in order
to solutions
simple
millions of years ago.
m discovered
(A) illustrate
a feature
ourenvironment
solar systematdiscovered
(C)
Mars
had
aof
wet
one time in the
17. Pasteur since
characterizes
“past
discourses
on spontaneous
thepast.
author’s
childhood
generation”
(line
9)
as
having
ave been
(B) cite(D)
an object
too that
small
scaleMars
to have
been rocks
The rock
fellinfrom
resembled
included
in
the
author’s
diorama
(A) demonstrated
theon
futility
of practical
scientific
found
the Antarctic
ice field.
wn moon
(C) draw
and our
own
studies
(E)a parallel
The rockbetween
that fellitfrom
Mars
hadmoon
very few
day’s children
(D) failed
contrast
the
scientific
curiosity
of
today’s
children
(B)
because
of
incomplete
knowledge
about
microscopic fissures.
with that ofofchildren
years ago
sterilization
apparatus
mmitment to
(E) enabled
emphasize
need for a greater
commitment
(C)
himthe
to understand
inconsistencies
in histo
space
exploration
early experiments
(D) failed to increase scientific knowledge
49-53
22.(E)
Theresolved
reasoningmuch
process
presented
in lines
49-53
of the
controversy
surrounding
the
(“As .issue
. . stars”) is best described as
y
observers
rvation
evidence
(A) inference based on an untested theory
18. The
stone” (lines
to the
(B)“little
extrapolation
from12-13)
similarrefers
situations
(C)
analysis
of
a
single
case
by
multiple
observers
(A) slight addition that Pasteur hoped to make
to
(D) hypothesis
confirmed
by
direct
observation
the existing body of facts
(E) small
comparison
of theory
physical
evidence
(B)
effect that
Pasteurwith
wanted
to have
on
one person’s learning
(C) minor disappointment Pasteur felt at being
rebuffed by his colleagues
(D) narrow-mindedness of those who cling to
scientific fallacies
(E) imperceptible progress that Pasteur had made
in understanding spontaneous generation
STOP
23.Pasteur’s
In lines 53-59
the author about
refers preparation
to the Goldilocks
fairy
19.
pronouncement
and chance
(“Yet
. . .implies
water”)that
in order to make which point
intale
lines
20-23
about a particular planet?
(A) only projects that have an immediate application
(A) The
planet’s environment may be conducive
are important
to a result
someascientists
arechances
eager tooffind.
(B) practice
improves
scientist’s
making
(B) The
planet’s atmosphere
a significant
discovery was once thought
be too cold
support
biological
life.useful
(C) fewtoscientists
aretolucky
enough
to devise
(C) The
simple methods astronomers used to
theories
discover
the features
of this
planet resem(D) work
on projects
that have
no immediate
applible theprepares
explorations
of curious
children.
cation
scientists
to exploit
chance
(D) Scientists’
discoverieswishful speculations about the
existence
of discoveries
this planet deserve
little
(E) most
scientific
that have
no more
immediate
credence than
a fairy
tale.
application
are the
result
of good luck and
(E) Only
after much trial and error did astronomers
timing
determine the precise location of this planet.
20. In context, the reference to the Academy of Sciences
24.(line
Which
the following,
true, would the “hopeful
35)of
serves
to suggestifwhy
scientists” (line 62) most likely interpret as evidence
(A)
Pasteur
wasfor
so determined
to make a significant
of the
potential
life on Mars?
contribution to scientific knowledge
(A) Pasteur
Mars was
by to
thereplicate
same planetary
collision
(B)
feltaffected
compelled
Spallanzani’s
that caused the extinction of dinosaurs.
experiments
(B) spontaneous
Mars had a very
mild atmospheric
(C)
generation
had already temperature
begun to
of years
bemillions
discredited
whenago.
Pasteur began his
(C) Mars
had a wet environment at one time in the
experimentation
past. believed he needed to design experiments
(D) Pasteur
(D) The
fell
from Mars resembled rocks
that rock
werethat
more
persuasive
found on the
Antarcticwas
iceviewed
field. by Pasteur’s
(E) spontaneous
generation
(E) The
rock
that
fell
from
Mars
very
colleagues as a topic that washad
unfit
forfew
scientific
microscopic fissures.
study
21. In line 41, “rich” most nearly means
(A)
(B)
(C)
(D)
(E)
precious
vital
abundant
meaningful
productive
22. The “conclusive experiment” (line 56) performed by
Pasteur was designed to answer critics who argued that
STOP
(A) the apparatus used in Pasteur’s earlier experiments had not been adequately sterilized
(B) Pasteur’s experiments related to spontaneous
generation had no immediate application
(C) the results of Pasteur’s experiments in the Alps
and in the cellar could not be replicated
(D) the broth in the flasks of Pasteur’s earlier experiments was not nutritious enough
(E) heating made the air in the flasks of the earlier
experiments unfit for spontaneous generation
is called, you may check your
work
on this
section
If you
finish
before
time only.
is called, you may check your work on this section only.
ot turn to any other section in the test.
Do not turn to any other section in the test.
-7-
23. The
In the
context
thephrase
passage
as new
arefers
whole,
24. In
In lines
his conclusive
kept the flasks
line 28) 19.
23.
In lines
the author
to the
the Goldilocks
author
usesof53-59
the
“this
intimacy”
(line 28) fairy 23.
53-59 the experiment,
author refersPasteur
to the Goldilocks
fairy
“vital
force”
(line
60)
is
best
described
as
vertical
(line
64)
in
order
to
(“Yet . . . water”) in order to make which point
to refer tale
to the
tale (“Yet . . . water”) in order to make which point
about
a particular
planet?
about
a particular
(A) hands-on
what
Pasteur
called
the
basic unitexperience
of life
(A) prevent
freshplanet?
air from entering them
e repre(A)
quality
of the
learning
repre(B) a sented
term
that
was
outdated
in
Pasteur’s
time
(B)
retain
the
boiling
liquid inside
flasks
(A) The
planet’s
environment
by the
shoebox
diorama may be conducive
(A) The planet’s environment
may the
be conducive
(C)
nutrients
necessary
for
sustaining
life
(C)
prevent
the
fluid
from
touching
trapped
bacteria
e about
to
a
result
some
scientists
are
eager
to
find.
(B) understanding that nonspecialists now have about
to a result some scientists are eager to find.
(D) that
which
thephenomena
power
to destroy
(D) The
avoidplanet’s
disturbing
the solution
(B)
Thehas
planet’s
atmosphere
waslife
once thought
meteorological
(B)
atmosphere
was inside
once thought
(E) general
what opponents
ofcold
Pasteur
believed
tobiological
be life.
(E) replicate
previous
experiments
exactly
gical
to be too
to theory
support
biological
(C)
acceptance
of the
that
to be toohis
cold
to support
biological
life.
alife
source
of
life
(C)once
Theexisted
simpleon
methods
Mars astronomers used to
(C) The simple methods astronomers used to
bout
the that
features
of thishave
planet
resem(D) increaseddiscover
knowledge
scientists
about
discover the features of this planet resemthe explorations of curious children.
our solarble
system
ble the explorations of curious children.
on
(D)
speculations
about
(E) way
thatScientists’
events onwishful
one planet
affect those
on the
(D) Scientists’ wishful speculations about the
another existence of this planet deserve little more
existence of this planet deserve little more
credence than a fairy tale.
credence than a fairy tale.
(E)“crude”
Only after
trial
and error did astronomers
(E) Only after much trial and error did astronomers
20. In line 33,
mostmuch
nearly
means
determine the precise location of this planet.
determine the precise location of this planet.
(A) natural and unaltered
(B) 24.
rough
andof
inexpert
Which
the following, if true, would the “hopeful
24. Which of the following, if true, would the “hopeful
(C) obvious
scientists” (line 62) most likely interpret as evidence
scientists” (line 62) most likely interpret as evidence
(D) vulgar
of the potential for life on Mars?
of the potential for life on Mars?
(E) nonspecific
(A) Mars was affected by the same planetary collision
(A) Mars was affected by the same planetary collision
that caused
extinction
of dinosaurs.
t likely 21. In line 42, the author
that caused the extinction of dinosaurs.
refersthe
to Pluto’s
moon
most likely
(B) Mars had a very mild atmospheric temperature
in order(B)
to Mars had a very mild atmospheric temperature
millions of years ago.
millions of years ago.
overed
(A) illustrate
a feature
ourenvironment
solar systematdiscovered
(C) Mars
had aof
wet
one time in the
(C) Mars had a wet environment at one time in the
since thepast.
author’s childhood
past.
en
(B) cite(D)
an object
too that
small
scaleMars
to have
been rocks
The rock
fellinfrom
resembled
(D) The rock that fell from Mars resembled rocks
includedfound
in theonauthor’s
diorama
the Antarctic
ice field.
found on the Antarctic ice field.
oon
(C) draw
a
parallel
between
it
and our
own
(E) The rock that fell from
Mars
hadmoon
very few
(E) The rock that fell from Mars had very few
children
(D) contrast the
scientific curiosity
microscopic
fissures. of today’s children
microscopic fissures.
with that of children years ago
ment to
(E) emphasize the need for a greater commitment to
space exploration
STOP
vers
n
nce
22. The reasoning process presented in lines 49-53
If you
finish
before
(“As . . . stars”)
is best
described
as time is called, you may check your work on this section only.
Do not turn to any other section in the test.
(A) inference based on an untested theory
(B) extrapolation from similar situations
(C) analysis of a single case by multiple observers
(D) hypothesis confirmed by direct observation
(E) comparison of theory with physical evidence
STOP
STOP
led, you may check your
work
on this
section
If you
finish
before
time only.
is called, you may check your work on this section only.
n to any other section in the test.
Do not turn to any other section in the test.
-8-
NO TEST MATERIAL ON THIS PAGE.
-9-
SECTION
SECTION33
SECTION 3
Time — 25 minutes
Time — 25 minutes
20 Questions
18 Questions
Time — 25 minutes
18 Questions
Turn to Section 3 (page 4) of your answer sheet Turn
to answer
the questions
section.
to Section
3 (page 4)inin
ofthis
your
answer sheet to answe
Turn to Section 3 (page 4) of your answer sheet to answer
the questions
this
section.
Directions: For this section, solve each problem and decide which is the best of the choices given. Fill in the corresponding
Directions:
This section
contains
two
typesFor
of questions
questions.1-8,
Yousolve
have 25 minutes to
Directions: This section contains two types of questions. You
have 25 minutes
to complete
both
types.
circle on the answer sheet. You may use any available space for scratchwork.
each
problem
and decide which
is the
bestanswer
of the sheet.
choicesYou
given.
each problem and decide which is the best of the choices given.
Fill
in the corresponding
circle
on the
mayFill in the corresp
use any available space for scratchwork.
use any available space for scratchwork.
1. Which of the following represents the total cost, in
1. Each month, a telephone service charges a base rate
of k compact
discsservice
at $15 charges
each anda base
p compact
1.dollars,
Each month,
a telephone
rate
of $10.00 and an additional $0.08 per call for the first
disc
cases atand
$25aneach?
(Disregard
tax.)for the first
of $10.00
additional
$0.08sales
per call
40 calls and $0.04 for every call after that. How much
40 calls and $0.04 for every call after that. How much
(A) 15k + 25 p
does the telephone service charge for a month in which
does the telephone service charge for a month in which
(B)
25k are
+ 15
p
50 calls are made?
50 calls
made?
(C) 40 k + p
(A) $12.20
(A) $12.20
(D) 0.40( k + p)
(B) $12.80
(B) $12.80
(E)
+ k 25 + p
(C) $13.60
(C) 15
$13.60
(D) $14.40
(D) $14.40
(E) $17.60
(E) $17.60
2. If the areas of the two rectangles in the figure above
are equal, which of the following could be the
coordinates of point R ?
2. According to the chart above, Company XYZ experienced−2its
increase in monthly profits between
(A)
, −largest
4
which
two
consecutive
months?
(B) −2, 2
(A)
(C)
(B)
(D)
(C)
(E)
(D)
January
−2
, 4 and February
February
and March
2, − 4
March and April
2April
, 4 and May
(E) May and June
- 10 -
2. Accordin
enced its
which tw
(A)
(B)
(C)
(D)
(E)
Jan
Feb
Ma
Apr
Ma
1
1
1
1
1
1
be which
3. A box contains 2,900 solid-colored marbles that
3
SECTION5.3If 6 + 7 + 8 > x + 7 + 8 , then x couldSECTION
are either orange, blue, or green. If 29 percent of Time — 25 minutes
Time — 25 minutes
of the following?
the marbles are orange and 29 percent of the marbles 18 Questions
18 Questions
are blue, what percent are green?
(A) 3
(B) 4
(A) Turn
29% to Section 3 (page 4) of your answer sheetTurn
to5Section
3 (page 4) in
of this
yoursection.
answer sheet to an
to(C)
answer
the questions
(B) 42%
(D) 6
(C) 52%
(E)
Directions:
This7 section
contains
two
typesFor
of questions
questions.1-8,
Yousolve
have 25 minu
Directions:
This
section
contains
two
types
of
questions.
You
have
25
minutes
to complete
both
types.
(D) 58%
each
problem
and
decide
which
is
the
best
of
the
choices
given.
Fill in the co
each
problem
and
decide
which
is
the
best
of
the
choices
given.
Fill
in
the
corresponding
circle
on
the
answer
sheet.
You
may
(E) 71%
use any available space for scratchwork.
use any available space for scratchwork.
P
Q
1, 3, 5, 6
2, 4, 6, 7, 9
6. If tx + 5 = (t + 1) x, which of the following must be
true?
4. Sets P and Q are shown above. If x is a member
of set P and y is a member of set Q, which of the
following CANNOT be equal to the product xy ?
(A) x = 4
(B) x = 5
1. Each month,
service charges a base rate
1. (A)
Each16month, a telephone service charges a base rate
4
(C) t a=telephone
(B)
18 and an additional $0.08 per call for the first
of $10.00
of $10.00
(D)and
t =an 5additional $0.08 per call for the first
(C)
20 and $0.04 for every call after that. How much
40 calls(E)
and t$0.04
40 calls
= 5 xfor every call after that. How much
(D)
does the telephone service charge for a month in which
does21
the telephone service charge for a month in which
(E)
24 are made?
50 calls are made?
50 calls
(A)
(B)
(C)
(D)
(E)
$12.20
$12.80
$13.60
$14.40
$17.60
(A)
(B)
(C)
(D)
(E)
$12.20
$12.80
$13.60
$14.40
$17.60
2. According to the chart above, Company XYZ experienced its largest increase in monthly profits between
which two consecutive months?
(A)
(B)
(C)
(D)
(E)
- 11 -
January and February
February and March
March and April
April and May
May and June
2. Ac
enc
wh
(A)
(B)
(C)
(D)
(E)
Questions 7-9 refer to the following definition.
Let # be defined by a # b
a and b.
ab
a
SECTION 3
3 x
7 x
2
k
SECTION 3
Time — 25 minutes
Time — 25 minutes
10. In the equation above, k is a constant. If the roots
b for all numbers18 Questions
18 Questions
of the equation are 7 and 2, what is the value of k ?
to 0Section
3 (page 4) in
of this
yoursection.
answer sheet to answe
Turn to Section 3 (page 4) of your answer sheetTurn
to(A)
answer
the questions
7.
(B) 2
(C) 3
Directions:
This7section
contains
two
typesFor
of questions
questions.1-8,
Yousolve
have 25 minutes to
Directions:
have 25 (D)
minutes
to complete
both
types.
(A) 7 This section contains two types of questions. You
each
problem
and
decide
which
is
the
best
of
the
choices
given.
Fill in the corresp
each
and decide which is the best of the choices given.
Fill
in the
corresponding
circle
on
the
answer
sheet.
You
may
(B)problem
10
(E) 14
use any available space for scratchwork.
use(C)
any17
available space for scratchwork.
(D) 20
(E) 32
2#5
8. If 10 # h
98, then h
(A) 8
(B) 9
(C) 10
(D) 11
(E) 12
11. Which of the following represents the area, A, of a
circle as a function of its diameter, d ?
(A)
A(d )
d
(B) A(d ) 2 d
1. Each month, a telephone service charges a base rate
1. Each month, a telephone service charges a base rate
of $10.00 and an additional $0.08 per call for the first
of $10.00 and an additional $0.08 per call for the first
40 calls
40 calls and $0.04 for every call after that. How much
A(d ) for every
d 2 call after that. How much
(C)and $0.04
does the telephone service charge for a month in which
does the telephone service charge for a month in which
50 calls are made?
50 calls are made?
d2
(D) A(d )
(A)
$12.20
(A) $12.20
2
(B) $12.80
(B) $12.80
d2
(C) $13.60
(C) $13.60
(E) A(d )
4
(D) $14.40
9. For
value of x is the statement x # y x
(D)what
$14.40
(E)
$17.60
(E) $17.60
always
true?
(A)
(B)
(C)
(D)
(E)
2
1
0
1
2
2. According to the chart above, Company XYZ experienced its largest increase in monthly profits between
which two consecutive months?
(A)
(B)
(C)
(D)
(E)
- 12 -
January and February
February and March
March and April
April and May
May and June
2. Accordi
enced its
which tw
(A)
(B)
(C)
(D)
(E)
Jan
Feb
Ma
Ap
Ma
x
f x
z
g z
4
5
5
4
8
a
10
b
SECTION 3
SECTION 3
Time — 25 minutes
18 Questions
Time — 25 minutes
18 Questions
to Section
3 (page 4) in
of this
yoursection.
answer sheet to an
Turn
3 (page
answer sheetTurn
to answer
the questions
12 to Section
9
20 4) of your
7
Directions:
This section
contains
two
typesFor
of questions
questions.1-8,
Yousolve
have 25 minu
Directions:
This section
two types
questions. You
have 25 minutes
to complete
both
types.
12.
The table above
shows contains
some values
for theoffunctions
each
problem
and
decide
which
is
the
best
of
the
choices
given.
Fill in the co
eachf problem
and
decide
which
is
the
best
of
the
choices
given.
Fill
in
the
corresponding
circle
on
the
answer
sheet.
You
may
and g . If f and g are linear functions, what
use any available space for scratchwork.
use any available space for scratchwork.
is the value of a b ?
(A)
(B)
(C)
(D)
(E)
0
1
2
3
4
13. The depth of a lake is the difference between the altitude
at the surface and at the lowest point of the lake. If the
five lakes in the graph above were listed in order from
the greatest depth to the least depth, which lake would
be third in the list?
(A)
(B)
(C)
(D)
(E)
Erie
Huron
Michigan
Ontario
Superior
1. Each month, a telephone service charges a base rate
1. Each month, a telephone service charges a base rate
of $10.00 and an additional $0.08 per call for the first
of $10.00 and an additional $0.08 per call for the first
40 calls and $0.04 for every call after that. How much
40 calls and $0.04 for every call after that. How much
does the telephone service charge for a month in which
does the telephone service charge for a month in which
50 calls are made?
50 calls are made?
(A)
(B)
(C)
(D)
(E)
$12.20
$12.80
$13.60
$14.40
$17.60
(A)
(B)
(C)
(D)
(E)
$12.20
$12.80
$13.60
$14.40
$17.60
2. According to the chart above, Company XYZ experienced its largest increase in monthly profits between
which two consecutive months?
(A)
(B)
(C)
(D)
(E)
- 13 -
January and February
February and March
March and April
April and May
May and June
2. Ac
enc
wh
(A
(B)
(C)
(D
(E)
SECTION
16.3If a number is chosen at random from the set SECTION 3
Time — 25 minutes
−12, − 6, 0, 6, 12 , what is the probability Time — 25 minutes
18 Questions that it is a member of the solution set of both 18 Questions
2 x − 3 < 7 and x + 5 > − 6 ?
to Section
3 (page 4) in
of this
yoursection.
answer sheet to answe
Turn to Section 3 (page 4) of your answer sheetTurn
to answer
the questions
(A) 0
Directions:
This1 section
contains
two
typesFor
of questions
questions.1-8,
Yousolve
have 25 minutes to
Directions: This section contains two types of questions. You
have 25 minutes
to complete
both
types.
(B)corresponding
each
problem
and5 decide which
is the
bestanswer
of the sheet.
choicesYou
given.
each problem and decide which is the best of the choices given.
Fill
in the
circle
on the
mayFill in the corresp
useIn
any
space xfor>scratchwork.
14.
theavailable
figure above,
90 and y = z + 1. If z is an use any available space for scratchwork.
2
(C)
integer, what is the greatest possible value of y ?
5
(A) 30
3
(D)
(B) 45
5
(C) 60
4
(D) 61
(E)
(E) 89
5
15. Molly is 64 inches tall. At 10:00 A.M. one day, her
shadow is 16 inches long, and the shadow of a nearby
tree is s inches long. In terms of s, what is the height,
17. If the length of AB is 5 and the length of BC is 6,
in inches, of the tree?
1. Each month,
service
charges
a base
rate
which aoftelephone
the following
could
be the
length
of AC ?
1. Each month, a telephone service charges a base rate
of $10.00 and an additional $0.08 per call for the first
(A)
s + 48and an additional $0.08 per call for the first
of $10.00
40 calls(A)
and 10
$0.04 for every call after that. How much
40 calls and $0.04 for every call after that. How much
(B)
12
does
the
telephone
service charge for a month in which
does
the
telephone
service
charge
for
a
month
in
which
(B) 2 s
13
50 calls(C)
are made?
50 calls are made?
s
(D) 15
(C)
(A) $12.20
(A) 4$12.20
(E) 16
(B) $12.80
(B) $12.80
(D)
(C) $13.60
(C) 4s$13.60
(D) $14.40
2
(D) $14.40
s
(E)
(E) $17.60
(E) $17.60
2
2. According to the chart above, Company XYZ experienced its largest increase in monthly profits between
which two consecutive months?
(A)
(B)
(C)
(D)
(E)
- 14 -
January and February
February and March
March and April
April and May
May and June
2. Accordin
enced its
which tw
(A)
(B)
(C)
(D)
(E)
Jan
Feb
Ma
Apr
Ma
SECTION 3
Time — 25 minutes
18 Questions
SECTION 3
Time — 25 minutes
18 Questions
to Section
3 (page 4) in
of this
yoursection.
answer sheet to an
Turn to Section 3 (page 4) of your answer sheetTurn
to answer
the questions
Directions:
This section
contains
two
typesFor
of questions
questions.1-8,
Yousolve
have 25 minu
Directions: This section contains two types of questions. You
have 25 minutes
to complete
both
types.
each
problem
and
is the
best
of the
choices
given.
each problem and decide which is the best of the choices given.
Fill
in the
circle
on the
sheet.
mayFill in the co
20.
Oncorresponding
the decide
numberwhich
line
above,
theanswer
tick
marks
areYou
equally
use any available
forofscratchwork.
use any available space for scratchwork.
spaced.space
Which
the lettered points represents y ?
18. In triangle ABC above, if AD = 6, DC = 3, and
(A) A
BC = 4, what is the area of triangle ABD ?
(B) B
(C) C
(D) D
(E) E
(A) 36
(B) 18
(C) 12
(D) 6
(E) 3
19. If x and y are two different integers and the
product 35xy is the square of an integer, which
of the following could be equal to xy ?
1. Each month, a telephone service charges a base rate
1. Each month, a telephone service charges a base rate
of $10.00 and an additional $0.08 per call for the first
(A)
5 and an additional $0.08 per call for the first
of $10.00
40 calls and $0.04 for every call after that. How much
(B)
70 and $0.04 for every call after that. How much
40 calls
does the telephone service charge for a month in which
(C)
does105
the telephone service charge for a month in which
50 calls are made?
(D)
140 are made?
50 calls
(E) 350
(A) $12.20
(A) $12.20
(B) $12.80
(B) $12.80
(C) $13.60
(C) $13.60
(D) $14.40
(D) $14.40
(E) $17.60
(E) $17.60
STOP
2. According
to on
the this
chartsection
above, Company
XYZ experiIf you finish before time is called, you may check
your work
only.
enced
its
largest
increase
in
monthly
profits
between
Do not turn to any other section in the test.
which two consecutive months?
(A) January and February
(B) February and March
(C) March and April
NOTE (D) April and May
(E) May
and June by the College Board.
In this exam, Section 4 did not count toward a score and was
not released
If you finish before time is called, you may check your work on this section only.
When you are instructed to begin the next section, omit Section 4 on your answer sheet and
Do not turn to any other section in the test.
mark your answers in the part of your answer sheet designated for the answers to Section 5.
STOP
- 15 -
2. Ac
enc
wh
(A)
(B)
(C)
(D)
(E)
SECTION 55
SECTION
Time —
— 25
25 minutes
minutes
Time
24 Questions
Questions
24
SECTION 5
Time — 25 minutes
24 Questions
Turn to
to Section
Section 55 (page
(page 5)
5) of
of your
your answer
answer sheet
sheet
to answer
answer
the questions
in your
this section.
section.
Turn
to Section
5questions
(page 5) of
answer sheet to answer t
Turn
to
the
in
this
eachthe
question
ingiven
this section,
best answer from among th
Directions: For
For each
each question
question in
in this
this section,
section, select
select the
the best
bestDirections:
answer from
fromFor
among
the
choices
given
and fill
fillselect
in the
thethe
corresponding
Directions:
answer
among
choices
and
in
corresponding
circle on the answer sheet.
circle on
on the
the answer
answer sheet.
sheet.
circle
3. The
In 1575
Venetians was
instituted
an annual reluctant
celebration
4.
representative
a traditionalist,
to to
Each sentence below has one or two blanks, each blank
------- the
of the ------that had with
struck
city.
support
anyend
legislation
inconsistent
thethe
nation’s
indicating that something has been omitted. Beneath
most ------- principles.
(A)five
lament
. . or
turmoil
the sentence are
words
sets of words labeled A
through E. Choose
the word or(B)
set. .of
words that, when
(B) orthodox
commemorate
plague
(A)
impassioned
(C) precarious
inserted in the(C)
sentence,
best
the meaning
of the
. .fits
pestilence
(D)eulogize
impressionable
(E) indeterminate
sentence as a(D)
whole.
hail . . prosperity
(E) author
solemnize
. . famea scenario in which playful,
5.
The
constructed
Example:
Example:
Example:
creative children are rewarded for their ------- and
Hoping4.to strict,
-------Horne’s
the dispute,
negotiators
proposed
Lena
singing
style is such
that she
can invest
dour
adults
are
punished
for their
-------.
Hoping to
to ------------- the
the dispute,
dispute, negotiators
negotiators proposed
proposed
Hoping
eventhat
thethey
mostfelt
------lyrics
with dramatic
a compromise
would
be ------to both meaning.
compromise that
that they
they felt
felt would
would be
be ------------- to
to both
both
aa compromise
(A) spontaneity . . rigidity
labor and management.
labor and
and management.
management.
labor
(A) digressions
harmonious. . mirth
(B) sensational
(B)
(A) enforce
. solemnity
useful
impeccable
(D) vapid
(C).(C)
. . malice
(A) enforce
enforce .. .. useful
useful
(A)
(B) end . .(D)
divisive
(E) esteemed
inflexibility
. . rigor
(B) end
end .. .. divisive
divisive
(B)
(C) overcome
. unattractive . . buoyancy
(E) .improvisations
(C) overcome
overcome .. .. unattractive
unattractive
(C)
(D) extend
. satisfactory theory about the migration of green
5. A. long-standing
(D) extend
extend .. .. satisfactory
satisfactory
(D)
(E) resolve
. . acceptable
turtles
wasusually
------- warm
by an and
innovative
marine
biologist
6. Although
------- in
greeting
friends,
(E) resolve
resolve .. .. acceptable
acceptable
(E)
Lauren
was too reserved
ever to be------truly by
-------.
who graciously
defused potential
dedicating
her work to the original researcher.
joyous . . conventional
our knowledge
of dinosaurs comes from
1.
of our knowledge
dinosaurs
comes
1. Much
After winning
the lottery,ofJohn
bought
sportsfrom
cars, built 1. Much of(A)
(B)
cordial
. . effusive
(A)
instigated
. in
. rancor
excavated bones,
which,
------- other clues such as
excavated
which,
in ------other
clues such
as a mansion,bones,
and wore
designer
suits;
however,
by thus
(C)
restrained
.
.
(B)
renounced
.
.gracious
approval
fossilized
tracks
and
eggs,
help
us to ------- the
fossilized
tracks he
andalienated
eggs, help
to ------- the
------ his -------,
hisus
friends.
dismissive
ebullient
(C)
displaced
. ...attribution
evolution(D)
of these
creatures.
evolution of these creatures.
(A) enduring . . hardship
(E)
. . antisocial
(D) genial
enhanced
. . alteration
(A) convergence
with . .. supplant
(A)
with . . supplant
(B) convergence
flaunting . . prosperity
(E) repudiated
. acrimony
(B) 7.
divergence
from . .argue
decode
(B)
from
. . decode
(C) divergence
undermining
. . image
Legal scholars
that when “justice” is interpreted
(C) dependence
on .the
. belie
(C)
. . belie
(D) dependence
calculating .on
. successes
too broadly,
concept becomes -------, easily changed
(D) opposition
to . . amplify
(D)
. amplify
(E) opposition
moderatingto. . consumption
and controlled
by outside forces.
(E) conjunction with . . trace
(E) conjunction with . . trace
(A) malleable
(B) influential
(C) coherent
2. The study of biology, once considered the key to
(D)arefelicitous
(E) prosaic
2.
Vernal
pools
among
the
most
------of
ponds:
they
2. Vernal
pools
are
among
the
most
------of
ponds:
they
solving nature’s mysteries, has instead served to
form as a result of snowmelt and a high water table in
form
as a result
of snowmelt
a high water table in
emphasize
nature’s
incredibleand
-------.
8. and
Thethen
instructor’s
voice
so ------- that most students
winter,
they ------by was
late summer.
winter, and then they ------- by late summer.
(A) tranquillity
(B) immobility
(C) consistency
preferred taking a test to listening to its grating sound.
(A) transitory . . expand
(A)(D)
transitory
. . expand
desirability
(E) complexity
(A) receptive
(B) cajoling
(C) melodious
(B) anachronistic
. . overflow
(B) anachronistic . . overflow
(D)
muted
(E)
strident
(C)
immutable
.
.
drain
(C) immutable . . drain
(D) itinerant . . teem
(D) itinerant . . teem
(E) ephemeral . . evaporate
(E) ephemeral . . evaporate
Each sentence
sentence below
below has
has one
one or
or two
two blanks,
blanks, each
each blank
blank
Each
indicating that
that something
something has
has been
been omitted.
omitted. Beneath
Beneath
indicating
the sentence
sentence are
are five
five words
words or
or sets
sets of
of words
words labeled
labeled A
A
the
through E.
E. Choose
Choose the
the word
word or
or set
set of
of words
words that,
that, when
when
through
inserted in
in the
the sentence,
sentence, best
best fits
fits the
the meaning
meaning of
of the
the
inserted
sentence as
as aa whole.
whole.
sentence
3. The ------- experiences of Madonna Swan, the 1983
North American Indian Woman of the Year, cannot be
fully appreciated if they are ------- in a tidy summary.
(A)
(B)
(C)
(D)
(E)
varied . . interposed
diverse . . condensed
profound . . magnified
transformative . . embellished
impressive . . immersed
3. The ------- experiences of Madonna Swan, the 1983
North American Indian Woman of the Year, cannot be
fully appreciated if they are ------- in a tidy summary.
(A)
(B)
(C)
(D)
(E)
- 16 -
varied . . interposed
diverse . . condensed
profound . . magnified
transformative . . embellished
impressive . . immersed
4. The repres
support any
most -------
(A) orthod
(D) imp
5. The author
creative ch
strict, dour
(A)
(B)
(C)
(D)
(E)
sponta
digres
solem
inflex
impro
6. Although u
Lauren wa
(A)
(B)
(C)
(D)
(E)
joyou
cordia
restra
dismi
genial
7. Legal scho
too broadly
and contro
(A) mallea
(D) feli
8. The instruc
preferred ta
(A) recept
(D) mut
SECTION
SECTION
5 the questions on the basis of what is stated
Each passage below is followed by questions based on its
content. Answer
or implied 5
Time
— 25 minutes
Time
—
25
minutes
in each passage and in any introductory material that may be provided.
24 Questions
24 Questions
Questions 6-7 are based on the following passage.
Questions 8-9 are based on the following passage.
Turn
to Section
5 (page 5) of
answer sheet to answ
Turn to Section 5 (page 5) of your answer sheet
to answer
the questions
in your
this section.
On the morning of June 13, 1998, a 4.6-billion-year-old
Apes raised by humans seem to pretend more frequently
eachthe
question
ingiven
this
section,
the
best answer from amo
Directions:
Forobject
each question
this
section,
select the and
bestDirections:
answer from
among
choices
andhandlers
fillselect
in the
corresponding
extraterrestrial
streaked in
into
Earth’s
atmosphere
thanFor
do apes
in
the
wild.
Animal
see
behaviors
the answer
sheet.
circle
the answer
blew on
to pieces
in thesheet.
sky somewhere in the neighborhood circle on they
interpret
as pretending practically every day. But Anne
Line of Nelda Wallace’s backyard. A dark basketball-size object
Line Russon, a psychologist, says she has found only about 20
4. The r
5 dropped with a loud ssshhht into Wallace’s garden, and
5 recorded
cases of possible
in free-ranging
4. The representative
was pretending
a traditionalist,
reluctant to
Each sentence below has one or two blanks, each blank
Each
sentence
below
hasproperties
one or two
blanks,
suppo
fragments
pelted
other
—only
theeach
firstblank
of many
orangutans,
culled
from thousands
of hours
support any
legislation
inconsistent
with of
theobservation.
nation’s
indicating that something has been omitted. Beneath
indicating
that something
has in
been
omitted.
Beneath are
most
strange things
soon to occur
town.
For meteorites
Onemost
possible
she noted in an e-mail interview from
-------reason,
principles.
the sentence are five words or sets of words labeled A
the
sentence
arestars
five of
words
or sets of words
labeled
A
more
than just
science-fiction
movies.
Scientists
her field station in Borneo, is that researchers have not been
(A) o
through E. Choose
the word or(B)
set of
words that, when
(A) orthodox
impassioned
(C) precarious
through
E. Choose
word scoop
or set of
words
that,
whenat
covet them,
privatethe
dealers
them
up for
resale
looking
for such behavior.
But many researchers
believe
(D
inserted in the sentence,
best fits the meaning
of the
(D)
impressionable
(E)
indeterminate
inserted
in
the
sentence,
best
fits
the
meaning
of
the
10 spiraling prices, and professional searchers travel the
10 that interaction with humans—and the encouragement to
sentence
as
a
whole.
sentence
a whole.
world toas
hunt
them down. Nelda Wallace’s town was
pretend that comes with it—may play a major role in why
5. The a
5. The authorapes
constructed
a scenario in which playful,
about
to
be
playact more.
Example:domesticated
Example: invaded by meteorite dealers, meteorite
creati
creative
children
are
rewarded
for
their
------and
fans, meteorite poachers, and other alien life-forms.
strict,
Hoping to strict,
-------dour
the dispute,
negotiators
adults are
punishedproposed
for their -------.
Hoping to ------- the dispute, negotiators proposed
8. Russon’s hypothesis would be most fully tested by
a compromise that they felt would be ------- to both
a
compromise
that
they
felt
would
be
------to
both
(A)
which
possible research
project?
6. The sentence in lines 1-4 (“On the morning . . .
(A)
spontaneity
. . rigidity
labor and management.
labor
and management.
(B)
backyard”)
is best characterized as
(B) digressions . . mirth
(A) Examining data from observations of pretending
(C)
(A) enforce
useful
(C). . solemnity
. . malice
(A)(A)
enforce
.
.
useful
ironic
behavior in apes other than orangutans
(D)
(B)
end
.
.
divisive
(D)
inflexibility
.
.
rigor
(B)(B)
enddramatic
. . divisive
(B) Expanding ongoing observations of orangutans
(E)
(C)
overcome
.
.
unattractive
(E)
improvisations
.
.
buoyancy
(C)(C)
overcome
comical. . unattractive
to include pretending behavior
(D) extend . . satisfactory
(D)(D)
extend
.
.
satisfactory
nostalgic
(C) Documenting pretending behavior among
(E) resolve
. . acceptable
6. Although
usually warm
in greeting friends, 6. Altho
(E)(E)
resolve
. . acceptable
celebratory
orangutans
raisedand
by ------humans
Laure
Lauren
was
too
reserved
ever
to
be
truly
-------. in
(D) Comparing specific pretending
behaviors
(A)
free-ranging
and domesticated
orangutans
7. The reference to the “alien life-forms” (line 13)
joyous
. . conventional
1. Much of(A)
our knowledge
of dinosaurs
comes from
1. Much of our knowledge of dinosaurs comes from
(B)
(E)
Reviewing
existing
data
on
free-ranging
primarily serves to
(B)
cordial
.
.
effusive
excavated bones, which, in ------- other clues such as
excavated bones, which, in ------- other clues such as
(C)
orangutans
to
determine
the
earliest
record
restrained
. . gracious
fossilized(C)
tracks
and eggs,
help us to ------- the
fossilized
and eggs,
help by
us some
to ------- the
(A) hint tracks
at the dangers
posed
(D)
of
pretending
behavior
(D)
dismissive
.
.
ebullient
evolution of these creatures.
evolution
of these creatures.
unexpected
visitors
(E)
(E) genial . . antisocial
(B) convergence
mock the public’s
(A) convergence
with . . supplant
(A)
with . fascination
. supplant with
9. Which theoretical
statement about pretending behavior
extraterrestrial
(B) 7.
divergence
from
decode
(B) divergence
from .beings
. decode
in apes
would. .argue
be
supported
most
fully byis the
“many 7. Lega
Legal
scholars
that when
“justice”
interpreted
(C)
indicate
the
dearth
of reliable information
(C) dependence
on
.
.
belie
too br
(C) dependence on
. . belie
researchers”
mentioned
in
line
9
?
too broadly, the concept becomes -------, easily changed
about a subject
(D) opposition
to . . amplify
and c
(D) opposition
to . . amplify
and controlled
by outside forces.
(D) conjunction
acknowledge
a lack
of familiarity with
(A) Having
ability to pretend has enabled
(E) conjunction
with the
. . trace
(E)
with
. . trace
(A) m
a scientific phenomenon
apes, such (B)
as chimpanzees,
be trained
(A) malleable
influential to(C)
coherent
(D
(E)
provide
a
humorous
label
for
a
certain
as
performers.
(D)
felicitous
(E)
prosaic
2. Vernal pools are among the most ------- of ponds: they 2. Vernal pools are among the most ------- of ponds: they
kind
of
zealotry
(B)
All
types
of
apes,
both
wild
and
domesticated,
form as a result of snowmelt and a high water table in
form as a result of snowmelt and a high water table in
can
with
human
companions.
8. and
Thethen
instructor’s
voice
was
so ------that most students 8. The i
winter,
theypretend
------by
late
summer.
winter, and then they ------- by late summer.
prefe
(C) Pretending
for wildtoapes
may vary
preferred
taking abehavior
test to listening
its grating
sound.
(A) transitory .considerably
. expand
(A) transitory . . expand
by region and population.
(A) r
(A)
cajoling apes
(C)domelodious
(B) anachronistic
. . overflow
(B) anachronistic . . overflow
(D) receptive
Handlers
of(B)
domesticated
not always
(D
(D)
muted
(E)
strident
(C)
immutable
.
.
drain
(C) immutable . . drain
have the rigorous observational training of
(D) itinerant . .scientists.
teem
(D) itinerant . . teem
(E) ephemeral
.
. evaporate
(E) ephemeral . . evaporate
(E) Wild
apes living apart from humans pretend
only rarely.
3. The ------- experiences of Madonna Swan, the 1983
3. The ------- experiences of Madonna Swan, the 1983
North American Indian Woman of the Year, cannot be
North American Indian Woman of the Year, cannot be
fully appreciated if they are ------- in a tidy summary.
fully appreciated if they are ------- in a tidy summary.
(A)
(B)
(C)
(D)
(E)
varied . . interposed
diverse . . condensed
profound . . magnified
transformative . . embellished
impressive . . immersed
(A)
(B)
(C)
(D)
(E)
- 17 -
varied . . interposed
diverse . . condensed
profound . . magnified
transformative . . embellished
impressive . . immersed
One summer my sister and I had toSECTION
take
Questions 10-18 are based on the following passage. SECTION everything.
5
5
ikebana,
the
art
of
flower
arrangement,
at
our
grandfather’s
Time — 25 minutes
Time — 25 minutes
50 school. The course was taught by Mrs. Oshima,24
a softThis excerpt is from a short story by a Japanese American
Questions
24 Questions
spoken, terrifying woman, and my supplies were provided
writer. The narrator reflects on her family’s past as she
by my grandmother, whose tastes ran to the oversized.
helps her mother prepare to move from her home.
Turn
to Section
5 that
(page
of
your
answer
sheet
to answer t
Turn to Section 5 (page 5) of your answer sheet
to
answer
theofquestions
inits
this
section.
I remember
little
class5)and
principles.
What
I
There’s a photograph of my mother standing on the pier
remember most clearly is having to walk home carrying
in Honolulu in 1932, the year she left Hawaii to attend the
55 from
oneFor
of
our
which,
more
often
not,
Directions:
eachcreations,
question
ingiven
this section,
select
besttowered
answer from among th
Directions:
For
each
question
in
this
section,
select
the
best
answer
among
the
choices
and fill
in than
thethe
corresponding
University of California. She’s loaded to the ears with leis.
above
our heads.
circle
on
the
answer
sheet.
circle
on
the
answer
sheet.
Line She’s wearing a fedora1 pulled smartly to the side. She is
How do we choose among what we experience, what
5 not smiling. Of my mother’s two years at the university,
we
are taught,
what wewas
runainto
by chance,reluctant
or what to
is
4. The repres
4. The
representative
traditionalist,
my grandmother
recalled
thatorshe
grades
Each sentence
below
has
one
or two
blanks,
eachofblank
Each
sentence below
has one
tworeceived
blanks, good
each blank
forced
upon
us?
What
is
the
principle
selection?
My
support any
support
any
legislation
inconsistent
with
the
nation’s
and neverthat
wore
a kimonohas
again.
secondBeneath
cousin, with indicating
that something
has
indicating
something
beenMy
omitted.
60 sisters
and
I areprinciples.
not been
boundomitted.
by any Beneath
of our mother’s oblimost ------most
------whom
my mother
she
arrived,
said
are five
sets ofthe
words
labeled
A
the
sentence
are fivestayed
wordswhen
or sets
offirst
words
labeled
A she the sentence
gations,
norwords
do weorfollow
rituals
that seemed
so
was surprisingly
—she
liked hats.
My mother through E.important.
(A) orthod
Choose
the
word
or(B)
set of
wordsdo
that,
(A) orthodox
impassioned
(C) precarious
through
E. Choosesophisticated
the word or set
of words
that, when
My
sister
once
asked,
youwhen
realize
that when
10 inserted
said that
she
was
homesick.
Her
favorite
class
was
(D) imp
the sentence,
best
thewas
meaning
ofabout
the how to make
(D)
(E)
indeterminate
in the sentence, best fits the meaning of the biology inserted inshe’s
goneimpressionable
that’s
it fits
? She
talking
and
she
entertained
thoughts
of
becoming
a
scientist.
Her
sentence
as
a
whole.
3
sentence as a whole.
sushi , but it was a more profound question nonetheless.
father, however, wanted her to become a teacher, and his
5. The author
5.
The author constructed a scenario in which playful,
Example:
wishes prevailed, even though he would not have forced Example:1 Acreative
creative ch
children
are
rewarded
for
their
------and
fedora is a soft felt hat popular in the United States in the 1930’s.
them
upon
Shethewas
a dutiful
daughter.proposed
strict, dour
Hoping2to
-------isdour
the
dispute,
negotiators
strict,
adults
punishedproposed
for their -------.
Hoping
toher.
------dispute,
negotiators
Chasuke
a rice
and teaare
mixture.
15
During her second
year,
livedbenear
campus
with a
a compromise
they
felt would
be cakes
-------and
tosometimes
both
3 Sushi isthat
a compromise
that they
feltshe
would
------to both
cold
rice
shaped
into
small
topped
or
(A) sponta
(A) spontaneity . . rigidity
mathematics
professor and his wife. In exchange for room labor andwrapped
management.
with garnishes.
labor and management.
(B) digres
(B)
digressions
.
.
mirth
and board she cleaned house, ironed, and helped prepare
(C) solem
(A)
enforce
.
.
useful
(C)
solemnity
.
.
malice
(A)
enforce
.
.
useful
meals. One of the things that survives from this period is a
(D) inflex
(B)
end
.
.
divisive
(D)
inflexibility
.
.
rigor
(B) end
. . divisive
black
composition
book entitled Recipes of California. As
10. The thematic focus of the passage is on the
(E) impro
(C)
overcome
.
.
unattractive
(E)
improvisations
.
.
buoyancy
overcome
. unattractive
20 a(C)
child,
I read it .like
a book of mysteries for clues to a life
(D)
extend
.
.
satisfactory
(A)
conflicts
between
the
narrator’s
mother
and
(D)
extend
.
.
satisfactory
both alien and familiar. Some entries she had copied by
(E) resolve
. . acceptable
grandmother
6. Although
usually warm and ------- in greeting friends, 6. Although u
(E) resolve
. . acceptable
hand;
others she
cut out of magazines and pasted on the
Lauren wa
(B) challenge
of balancing
values and
Lauren
was too reserved
ever conflicting
to be truly -------.
page, sometimes with a picture or drawing. The margins
practices
contained her cryptic comments: “Saturday bridge club,”
(A) joyou
joyous
. . conventional
1. Much of(A)
our
of assimilation
dinosaurs comes
from
Much
of our
dinosaurs comes from
(C)knowledge
widespread
of immigrants
into the
25 1.
“From
Mary
G. knowledge
Do not giveofaway.”
(B) cordia
(B)
cordial
.
.
effusive
excavated bones,culture
which,ofinthe
------otherStates
clues such as
excavated
which,
in ------clues
as
United
That book bones,
holds part
of the
answerother
to why
oursuch
family
(C) restra
restrained
. . gracious
fossilized(C)
tracks
and eggs,
help
us to -------traditions
the
fossilized
eggs,
help
------- the
(D)
desirability
of
maintaining
rituals
didn’t tracks
fit the and
norm
either
of us
ourtorelatives
or of the
(D) dismi
(D)
dismissive
.
.
ebullient
evolution (E)
of these
creatures.
evolution
of these
creatures.
irrelevance
of traditional customs to modern
larger
community
in which
we grew up. At home, we ate
(E) genial
(E) genial . . antisocial
society
in fear
the glass ofwith
spilled
milk, the stray elbow on the
(A) convergence
with . . supplant
(A) ofconvergence
. . supplant
30 table,
boarding house
At my grandparents’, we
(B) 7.
divergence
from . .argue
decode
(B)thedivergence
from . reach.
. decode
Legal scholars
that when “justice” is interpreted 7. Legal scho
11. The grandmother’s
comments in lines 5-7 imply that
(C) dependence
on
.
.
belie
slurped
our chasuke2on
. We
wore tailored dresses and black
too broadly
(C) dependence
. . belie
too broadly, the concept becomes -------, easily changed
her daughter’s
experiences at the university were
(D) opposition
to . . amplify
shoes
white socks;
however, what we longed for were
and contro
(D)with
opposition
to . . amplify
and controlled
by outside forces.
characterized
(E) conjunction
with . .by
trace
the(E)
lacy conjunction
colorful dresses
other girls wore to church on
withthat
. . trace
(A) mallea
(A) malleable
(B) influential
(C) coherent
Sunday. For six years, I marched to Japanese language
(A) success and camaraderie
(D) feli
(D)
felicitous
(E)
prosaic
among the mostand
------of ponds: they
35 2.
school
after
my regular
classes;
however,
spoke
Vernal
pools
are among
the most
-------we
of only
ponds:
they 2. Vernal pools
(B) are
accomplishment
assimilation
form as a (C)
resultenlightenment
of snowmelt and
high water table in
form at
as home.
a resultWe
of talked
snowmelt
a high
table in
English
too and
loudly
and water
all at once,
andaintrospection
8. and
The
instructor’s
voice
was
so ------- that most students 8. The instruc
then
they ------by
late summer.
winter,
and then
------which
mortified
my they
mother,
butby
shelate
wassummer.
always complaining winter,
(D)
diligence
and
homesickness
preferred ta
preferred
taking
a
test
to
listening
to its grating sound.
about Japanese indirectness. I know that she smarted under
(E) scholarship
(A) transitory
. . expand and competition
(A) transitory . . expand
a system in which the older son is the center of the familial
(A) recept
(A) receptive
(B) cajoling
(C) melodious
(B) anachronistic
. . overflow
(B) anachronistic . . overflow
40 universe, but at thirteen I had a fit of jealous rage over her
(D) mut
12.
In
line
11,
the
word
“entertained”
most
nearly
means
(D)
muted
(E)
strident
(C)
immutable
.
.
drain
(C) immutable . . drain
fawning attention to our only male cousin.
(D) itinerant
. teem
(D) itinerant . . teem
(A) . regaled
My sister has found a photograph of our mother, a
(E) ephemeral
. . evaporate
(E) ephemeral . . evaporate
(B) hosted
round-faced and serious twelve or thirteen, dressed in a
flaunted
kimono
and seated,
on herof
knees,
on theSwan,
tatamithe
mat.
She is 3. The -------(C)
experiences of Madonna Swan, the 1983
3.
The ------experiences
Madonna
1983
(D) harbored
45 playing the koto, a difficult stringed instrument thought to
North American Indian Woman of the Year, cannot be
North American Indian Woman of the Year, cannot be
(E) welcomed
teach
girls
discipline.ifOf
course,
everything
Japanese
was
fully appreciated if they are ------- in a tidy summary.
fully
appreciated
they
are ------in a tidy
summary.
a lesson in discipline—flower arranging, embroidery,
(A) varied . . interposed
(A) varied . . interposed
(B) diverse . . condensed
(B) diverse . . condensed
(C) profound . . magnified
(C) profound . . magnified
(D) transformative . . embellished
(D) transformative . . embellished
(E) impressive . . immersed
(E) impressive . . immersed
- 18 -
13. The narrator’s statement in line 14 (“She . . .
daughter”) serves to
childhood 5
SECTION17.
5 In lines 53-56, the narrator’s description ofSECTION
a sense
— of
25 minutes
Time — 25 minuteswalks home from ikebana class conveysTime
24 Questions
24 Questions (A) adventure
(A) defend her mother’s interest in science
(B) justify her mother’s decision to leave home
(B) relief
(C)
explain
why her mother
became
(C)
melancholy
Turn
to Section
5 (page 5) of
answer sheet to answ
Turn
to Section
5 (page
5) ofa teacher
your answer sheet
to answer
the questions
in your
this section.
(D) question the relevance of established customs
(D) absurdity
(E) rationalize her grandfather’s actions
(E) vitality
eachthe
question
this section,
best answer from amo
Directions: For each question in this section, select the bestDirections:
answer fromFor
among
choicesingiven
and fillselect
in thethe
corresponding
circle on the answer sheet.
circle
on
the
answer
sheet.
14. The narrator suggests that as a child she read her
18. To the narrator, her sister’s question (lines 62-63)
mother’s book of recipes in order to
implies that the
4. The r
4. The representative was a traditionalist, reluctant to
Each
sentence
below
has one
or two blanks,
each
blank
Each (A)
sentence
onemother’s
or two blanks,
each
blank
seekbelow
proof has
of her
devotion
to the
family
(A)
mother
represented
the last
true
vestige
of the
suppo
support
any legislation
inconsistent
with
the nation’s
something
hasJapanese
been omitted.
Beneath
indicating
that something
been
Beneathin her indicating that
(B) understand
morehas
fully
theomitted.
contradictions
sisters’
heritage
most
most ------principles.
the sentence are
wordsshould
or setshave
of words
the sentencemother’s
are five words
or sets of words labeled A
behavior
(B)five
mother
madelabeled
more ofA an effort to
(A) o
through E. Choose
the
word or
set
of
wordsabout
that, when
(A) orthodox
(B)
impassioned
(C)background
precarious
through
Choose thethe
word
or setshe
of created
words that,
(C)E.perpetuate
fantasy
aboutwhen
her
educate
her
daughters
their
(D
inserted in the(C)
sentence,
besteducation
fits the meaning
of the extended beyond
(D) mother’s
impressionable
(E)California
indeterminate
inserted in the
sentence, best fits the meaning of the
mother
in
sentence
as
a
whole.
sentence
a whole.
(D) assearch
for clues to her mother’s reluctance to
the confines of the university
5.
The
author
constructed
a scenario
in mother’s
which playful,
discuss
her
past
(D)
sisters
were saddened
by their
decision 5. The a
Example:
Example:
creati
creative
children
are
rewarded
for
their
------and
(E) discover the cause of her mother’s unhappiness
to move
strict,
Hoping to strict,
------the
dispute,
negotiators
proposed
dour
adults
are
punished
for
their
-------.
Hoping to ------- the dispute, negotiators proposed
(E) sisters would not regret the absence of traditional
a compromise thatfamily
they felt
would be ------- to both
a compromise
that they
wouldof
bethe
------to both
rituals
15.
The description
of thefelt
reaction
mother
to her
(A)
(A) spontaneity . . rigidity
labor
and management.
children’s
manner of speaking (lines 36-38) highlights labor and management.
(B)
(B) digressions . . mirth
she . . useful
(C)
(A) enforce
useful
(C). . solemnity
. . malice
(A)how
enforce
(D)
(B)
end
.
.
divisive
(D)
inflexibility
.
.
rigor
(B)(A)
endfeared
. . divisive
that her children’s naïveté would invite
(E)
(C)
overcome
.
.
unattractive
(E)
improvisations
.
.
buoyancy
(C) overcome
. . unattractive
trouble
(D)
extend
.
.
satisfactory
(D)(B)
extend
.
.
satisfactory
shared her children’s distaste for Japanese
(E) resolve
. . acceptable
6. Although
usually warm and ------- in greeting friends, 6. Altho
(E) resolve
. . acceptable
language
lessons
Laure
Lauren was too reserved ever to be truly -------.
(C) was still imbued with the lessons of her culture
(D) insisted on maintaining a strong Japanese
(A)
joyous . . conventional
1. Much of(A)
our knowledge
of dinosaurs comes from
1. Much ofinfluence
our knowledge
of dinosaurs comes from
in her home
(B)
(B)
cordial
.
.
effusive
excavated bones, which, in ------- other clues such as
excavated
bones,
otherinclues
such as
(E) wanted
her which,
childreninto------be fluent
the Japanese
(C)
restrained
. . gracious
fossilized(C)
tracks
and eggs,
help us to ------- the
fossilized
tracks and eggs, help us to ------- the
language
(D)
(D)
dismissive
.
.
ebullient
evolution of these creatures.
evolution of these creatures.
(E)
(E) genial . . antisocial
(A) convergence with . . supplant
16.(A)
Theconvergence
narrator repeats
“everything” in
withthe
. . word
supplant
linesdivergence
46-48 to from . . decode
(B) 7.
divergence
from . .argue
decode
(B)
Legal scholars
that when “justice” is interpreted 7. Lega
(C) dependence
on
.
.
belie
too br
(C)
dependence
on
.
.
belie
too broadly, the concept becomes -------, easily changed
(A) explain the intensity of her competition with her
(D) opposition
to . . amplify
and c
(D) opposition
to
.
.
amplify
and
controlled
by
outside
forces.
sister
(E) conjunction with . . trace
(E)
conjunction
with
.
.
trace
(B) characterize the diverse achievements of Japanese
(A) m
(A) malleable
(B) influential
(C) coherent
Americans
(D
(D)
felicitous
(E)
prosaic
2. Vernal pools are among the most ------- of ponds: they 2. Vernal pools are among the most ------- of ponds: they
(C) describe her mother’s single-minded pursuit of
form as a result of snowmelt and a high water table in
form as a result of snowmelt and a high water table in
perfection
8. and
Thethen
instructor’s
voice
so ------- that most students 8. The i
winter,
they ------by was
late summer.
winter, and then they ------- by late summer.
(D) emphasize the extent to which discipline governed
prefe
preferred taking a test to listening to its grating sound.
(A) transitory . . expand
Japanese
(A) transitory
. . life
expand
(A) r
(A) receptive
(B) cajoling
(C) melodious
(B) anachronistic
. . overflow
(E) anachronistic
highlight the.extraordinary
(B)
. overflow skill required to master
(D
(D)
muted
(E)
strident
(C)
immutable
.
.
drain
the
koto
(C) immutable . . drain
(D) itinerant . . teem
(D) itinerant . . teem
(E) ephemeral . . evaporate
(E) ephemeral . . evaporate
3. The ------- experiences of Madonna Swan, the 1983
North American Indian Woman of the Year, cannot be
fully appreciated if they are ------- in a tidy summary.
(A)
(B)
(C)
(D)
(E)
varied . . interposed
diverse . . condensed
profound . . magnified
transformative . . embellished
impressive . . immersed
3. The ------- experiences of Madonna Swan, the 1983
North American Indian Woman of the Year, cannot be
fully appreciated if they are ------- in a tidy summary.
(A)
(B)
(C)
(D)
(E)
- 19 -
varied . . interposed
diverse . . condensed
profound . . magnified
transformative . . embellished
impressive . . immersed
serve to 5
Questions 19-24 are based on the following passage. SECTION19.
SECTION
5 The first two paragraphs (lines 1-37) primarily
Timein—
25 minutes
Time — 25 minutes(A) argue against the depiction of children
artwork
The following passage is excerpted from a historian’s 24 Questions
Questions
(B) suggest that medieval Western art was24
particularly
examination of European attitudes toward childhood.
conservative
(C)
describe
unrealistic
portrayal
of children
in
Medieval
European
art until
about 5)
theof
twelfth
Turn
to Section
5 (page
5) of
your
answer
sheet
to answer t
Turn to
Section
5 (page
yourcentury
answer sheet
to answer
the the
questions
in
this section.
medieval art
did not know childhood or did not attempt to portray it.
(D) trace the evolution of realistic representation in
It is hard to believe that this neglect was due to incompeeach
question
this section,
best answer from among th
For eachitquestion
in this
section,that
select
thewas
bestDirections:
answer fromFor
among
the
choices
and fillselect
in thethe
corresponding
Western
artingiven
Line Directions:
tence or incapacity;
seems more
probable
there
circle
on
the
answer
sheet.
circle
on
the
answer
sheet.
(E) postulate a theory about the thematic focuses of
5 no place for childhood in the medieval world. A miniature
medieval Western art
painted during the twelfth century provides us with a
4. The repres
4. The representative was a traditionalist, reluctant to
striking
example
of
the
deformity
that
an
artist
at
that
time
Each
sentence
below
has one or two blanks, each blank
Each sentence below has one or two blanks, each blank
support any
support
any legislation
inconsistent
with the of
nation’s
would
inflict
on
the
representation
of
children’s
bodies.
20.
The
author’s
argument
about
the
depiction
children
indicating that something has been omitted. Beneath
indicating that something has been omitted. Beneath
most ------most
------- principles.
in
medieval
art
assumes
that
the
depictions
The
subject
is
a
Biblical
scene
in
which
Jesus
is
surrounded
the sentence are five words or sets of words labeled A
the sentence are five words or sets of words labeled A
10 by little children. Yet the miniaturist has grouped around through E. Choose
(A) orthod
the wordthe
or(B)
set of
wordsbetween
that, when
(A)
impassioned
(C)
precarious
through E. Choose the word or set of words that, when
(A) orthodox
suggest
connection
medieval
art and
Jesus what
aresentence,
obviously
eight
without of
anythe
charac- inserted in the sentence,
(D) imp
best fits the meaning
of the
(D) impressionable
(E) indeterminate
inserted
in the
best
fitsmen,
the meaning
religion
teristics
of
childhood;
they
have
simply
been
depicted
on
sentence
as
a
whole.
sentence as a whole.
(B) prefigure the gradual shift to realism
a smaller scale. In a French miniature of the late eleventh
5. The author
5.
The
constructed
a scenario
which
playful,
(C)author
are too
varied to support
anyinone
argument
Example:
century, three children brought to life by a saint are also Example: creative children are rewarded for their ------- and
creative ch
(D) reflect earlier civilizations’ corruption
15 reduced to a smaller scale than the adults, without any
strict, dour
Hoping to strict,
------the dispute,
negotiators
proposed
dour
adults
are
punished
for theirheld
-------.
Hoping to ------- the dispute, negotiators proposed
(E) offer
an indication
of commonly
attitudes
other difference in expression or features. A painter would a compromise that they felt would be ------- to both
a compromise that they felt would be ------- to both
(A) sponta
(A) spontaneity . . rigidity
not hesitate to give the body of a child the musculature of
labor and
labor and management.
21.management.
Thedigressions
author’s argument
(B) digres
(B)
. . mirthis developed primarily by
an adult.
(C) solem
(A) enforce
. . solemnity
useful
(C)
malice
(A)
enforce
In the
world. .ofuseful
pictorial formulas inherited from
(A)
quotations. .from
literary sources
(D) inflex
(B)
end
.
.
divisive
(D)
inflexibility
.
.
rigor
(B) end
. . divisive
20 ancient
Rome,
right up to the end of the thirteenth century,
(B) descriptions of visual evidence
(E) impro
(C)
overcome
.
.
unattractive
(E)
improvisations
.
.
buoyancy
(C) are
overcome
. . unattractive
there
no children
characterized by a special expression,
(C) psychological analyses of medieval artists
(D) extend(D)
. . satisfactory
(D)only
extend
. . on
satisfactory
but
adults
a reduced scale. This refusal to accept
comparisons of modern and medieval images
* in art is to be found too in most of the
(E) resolve
. . acceptable
6. Although
(E) morphology
resolve . . acceptable
of usually
the bodywarm and ------- in greeting friends, 6. Although u
child
Lauren wa
Lauren
was too reserved
ever to be truly
-------.
(E) reflections
on the philosophical
nature
of
ancient civilizations. A fine Sardinian bronze of the ninth
childhood
25 century B.C. shows a mother holding in her arms the bulky
(A) joyou
joyous
. . conventional
1. Much of(A)
our knowledge
of dinosaurs comes from
1.
Much
of son.
our knowledge
of catalog
dinosaurs
comes
fromlittle
body
of her
The museum
tells
us: “the
(B)
cordial
.
.
effusive
excavated
bones,
which,
in of
------other clues
such
as primarily (B) cordia
excavated
bones,
which,
in ------clues
such as
masculine
figure
could
also be
a childother
which,
in accor22.(C)
The
last
sentence
the
passage
(lines
46-50)
(C) restra
restrained
. . gracious
fossilizedserves
tracks
and eggs,
help us to ------- the
fossilized
and adopted
eggs, help
us to ------theby other
dance
with thetracks
formula
in ancient
times
to
(D) dismi
(D)
dismissive
.
.
ebullient
evolution of these creatures.
evolution
of these
creatures.as an adult.” Everything in
peoples,
had been
represented
(E) genial
(E)
genial
.
.
antisocial
(A) define an important term that is central to the
30 fact(A)
would
seem to suggest
the realistic representation
(A) convergence
with
.
.
supplant
convergence
with . that
. supplant
author’s argument
of children
or the idealization
of childhood was confined
(B) 7.
divergence
from . .argue
decode
(B) divergence
from . . decode
Legal
scholars
that to
when
“justice”thesis
is interpreted 7. Legal scho
(B)
dismiss
objections
the author’s
to ancient
Greek art. on
Representations
of Eros, the Greek
(C) dependence
on
.
.
belie
too broadly
(C) dependence
. . belie
too
theanconcept
becomes
easily changed
(C)broadly,
provide
explanation
for -------,
the phenomenon
child
of love, proliferated
in that Hellenistic period,
(D) opposition
to . . amplify
and contro
(D)godopposition
to . . amplify
and controlled
by
outside
forces.
discussed in the previous paragraphs
but(E)
childhood
disappeared
(E) conjunction with . . trace
conjunction
with . .from
traceart together with the other
(D) malleable
introduce examples
from other(C)
timecoherent
periods and
(A) mallea
(A)
(B) influential
35 Hellenistic themes, and the subsequent Romanesque art
other
forms
of
representational
art
(D) feli
(D)
felicitous
(E)
prosaic
returned
the rejection
of the
features
of
are among the most ------- of ponds: they
2.
Vernaltopools
are among
the special
most ------of ponds:
they 2. Vernal pools
(E)
summarize
the
views
of
other
historians
of
childhood.
form as a result of snowmelt and a high water table in
form as a result of snowmelt and a high water table in
medieval
artby was
8. and
Thethen
instructor’s
voice
so ------- that most students 8. The instruc
winter,
they ------late summer.
This
is no
Our
point in
winter,
andmere
thencoincidence.
they ------- by
latestarting
summer.
preferred ta
preferred taking a test to listening to its grating sound.
this study is a world of pictorial representation in which
(A) transitory . . expand
(A)
transitory
.
.
expand
40 childhood is unknown; literary historians such as Calvé
(A) recept
(A) receptive
(B) cajoling
(C) melodious
(B) anachronistic
. . overflow
(B)made
anachronistic
. . overflowabout the medieval epic,
have
the same observation
(D) mut
(D)
muted
(E)
strident
(C)
immutable
.
.
drain
(C) immutable
. . drain
in which
child prodigies
behave with the courage and
(D) itinerant . . teem
(D) itinerant
. teem
physical
strength .of
doughty warriors. This undoubtedly
(E) ephemeral . . evaporate
(E)
ephemeral
.
.
evaporate
meant that the people of the tenth and eleventh centuries
45 did not dwell on the image of childhood and that the
3. The ------- experiences of Madonna Swan, the 1983
3.
Thehad
------experiences
of Madonna
Swan,
1983
image
neither
interest nor
even reality
for the
them.
It
North American Indian Woman of the Year, cannot be
North
American
Indian
Woman
of
the
Year,
cannot
be
suggests too that in the realm of real life, and not simply
fully appreciated if they are ------- in a tidy summary.
fully
appreciated
if
they
are
------in
a
tidy
summary.
in that of aesthetic translation, childhood was a period of
(A) varied . . interposed
transition
that passed
quickly and that was just as quickly
(A) varied
. . interposed
(B) diverse . . condensed
50 forgotten.
(B) diverse . . condensed
(C) profound . . magnified
(C) profound . . magnified
* Structure
(D) transformative . . embellished
and form
(D) transformative
. . embellished
(E) impressive . . immersed
(E) impressive . . immersed
- 20 -
23. In line 48, “translation” most nearly means
way that 5
SECTION24.
5 The author offers which explanation for theSECTION
Time — 25 minutesmedieval painters depicted children? Time — 25 minutes
(A) substitution
24 Questions
24 Questions (A) Children were discouraged from becoming
(B) explanation
artists’
models.
(C) representation
(B)
Children
were
more5)
difficult
to section.
paint
than adults.
(D)
transportation
Turn
to Section
5
(page
of
answer
sheet to answ
Turn
to Section 5 (page 5) of your answer sheet
to answer
the questions
in your
this
(E) correction
(C) Children had never been a subject of art in
Western traditions.
each
question
ingiven
this
best answer
Directions: For each question in this section, select the bestDirections:
answer fromFor
among
the
choices
and fillselect
in the
(D)
Childhood
was
not section,
understood
as the
acorresponding
separate
phasefrom amo
circle on the answer of
sheet.
circle on the answer sheet.
life.
(E) Childhood was not recognized in medieval
4. The r
4. The representative was a traditionalist, reluctant to
Each sentence belowtheology.
has one or two blanks, each blank
Each sentence below has one or two blanks, each blank
suppo
support any legislation inconsistent with the nation’s
indicating that something has been omitted. Beneath
indicating that something has been omitted. Beneath
most
most ------- principles.
the sentence are five words or sets of words labeled A
the sentence are five words or sets of words labeled A
(A) o
through E. Choose
the word or(B)
set of
words that, when
(A) orthodox
impassioned
(C) precarious
through E. Choose the word or set of words that, when
(D)
inserted in the sentence,
best fits the meaning
of the
(D) impressionable
(E) indeterminate
inserted in the sentence, best fits the meaning of the
sentence
as
a
whole.
sentence as a whole.
5. The a
Example:5. The author constructed a scenario in which playful,
Example:
creati
creative children are rewarded for their ------- and
strict,
Hoping to strict,
-------dour
the dispute,
negotiators
adults are
punishedproposed
for their -------.
Hoping to ------- the dispute, negotiators proposed
a compromise that they felt would be ------- to both
a compromise that they felt would be ------- to both
(A) s
(A) spontaneity . . rigidity
labor and management.
labor and management.
(B) d
(B) digressions . . mirth
(C) s
(A) enforce
useful
(C). . solemnity
. . malice
(A) enforce . . useful
(D) i
(B)
end
.
.
divisive
(D)
inflexibility
.
.
rigor
(B) end . . divisive
(E) i
(C)
overcome
.
.
unattractive
(E)
improvisations
.
.
buoyancy
(C) overcome . . unattractive
(D) extend . . satisfactory
(D) extend . . satisfactory
(E) resolve
. . acceptable
6. Although
usually warm and ------- in greeting friends, 6. Altho
(E) resolve . . acceptable
Laure
Lauren was too reserved ever to be truly -------.
(A) j
joyous . . conventional
1. Much of(A)
our knowledge
of dinosaurs comes from
(B) c
(B)
cordial
.
.
effusive
excavated bones, which, in ------- other clues such as
(C) r
restrained
. . gracious
fossilized(C)
tracks
and eggs,
help us to ------- the
(D) d
(D)
dismissive
.
.
ebullient
evolution of these creatures.
(E) g
(E) genial . . antisocial
(A) convergence with . . supplant
(A) convergence with . . supplant
(B) 7.
divergence
from . .argue
decode
(B) divergence from . . decode
Legal scholars
that when “justice” is interpreted 7. Legal
(C) dependence
on
.
.
belie
too br
(C) dependence on . . belie
too broadly, the concept becomes -------, easily changed
(D) opposition
to . . amplify
and c
(D) opposition to . . amplify
and controlled
by outside forces.
(E) conjunction with . . trace
(E) conjunction with . . trace
(A) m
If you finish before time is called, you may check
work on(B)
this
section only.
(A)your
malleable
influential
(C) coherent
(D)
(D)
felicitous
(E)
prosaic
Do
not
turn
to
any
other
section
in
the
test.
2. Vernal pools are among the most ------- of ponds: they 2. Vernal pools are among the most ------- of ponds: they
form as a result of snowmelt and a high water table in
form as a result of snowmelt and a high water table in
8. and
Thethen
instructor’s
voice
so ------- that most students 8. The in
winter,
they ------by was
late summer.
winter, and then they ------- by late summer.
prefer
preferred taking a test to listening to its grating sound.
(A) transitory . . expand
(A) transitory . . expand
(A) r
(A) receptive
(B) cajoling
(C) melodious
(B) anachronistic
. . overflow
(B) anachronistic . . overflow
(D)
(D)
muted
(E)
strident
(C)
immutable
.
.
drain
(C) immutable . . drain
(D) itinerant . . teem
(D) itinerant . . teem
(E) ephemeral . . evaporate
(E) ephemeral . . evaporate
1. Much of our knowledge of dinosaurs comes from
excavated bones, which, in ------- other clues such as
fossilized tracks and eggs, help us to ------- the
evolution of these creatures.
STOP
3. The ------- experiences of Madonna Swan, the 1983
North American Indian Woman of the Year, cannot be
fully appreciated if they are ------- in a tidy summary.
(A)
(B)
(C)
(D)
(E)
varied . . interposed
diverse . . condensed
profound . . magnified
transformative . . embellished
impressive . . immersed
3. The ------- experiences of Madonna Swan, the 1983
North American Indian Woman of the Year, cannot be
fully appreciated if they are ------- in a tidy summary.
(A)
(B)
(C)
(D)
(E)
- 21 -
varied . . interposed
diverse . . condensed
profound . . magnified
transformative . . embellished
impressive . . immersed
SECTION
66
SECTION
Time
——
2525
minutes
Time
minutes
1818
Questions
Questions
SECTION 6
Time — 25 minutes
18 Questions
Turn
toto
Section
6 (page
6)6)
ofof
your
answer
sheet
to
answer
the
questions
this
Turn
to Section
6 to
(page
6) of
your
answerinsheet
tosection.
answer the questions
Turn
Section
6 (page
your
answer
sheet
answer
the
questions
in
thissection.
Directions:
section
contains
two
types
of of
questions.
have
2525
minutes
complete
both
types.
questions
1-8,
Directions:
ThisYou
section
contains
twototypes
of questions.
YouFor
have
25
minutes
tosolve
complete
Directions:This
This
section
contains
two
types
questions.
You
have
minutes
to
complete
both
types.
For
questions
1-8,
solve both types
each
problem
and
decide
which
is
the
best
of
the
choices
given.
Fill
in
the
corresponding
circle
on
the
answer
sheet.
You
may
each
problem
and
decide
which
is
the
best
of
the
choices
given.
Fill
in
the
corresponding
each problem and decide which is the best of the choices given. Fill in the corresponding circle on the answer sheet. You may circle on the
useuse
anyany
available
space
forfor
scratchwork.
available
space
scratchwork.use any available space for scratchwork.
1. A new roll of film has p pictures. After t pictures are
2. The prime number p is a fac
A new
film hastriples
p pictures.
pictures are
p is a factor
of for
30 an
andadult’s
is also
prime number
a, b, c After
does tNOT
2. 2.AnThe
amusement
park charges
$7 more
1. 1.Which
of roll
the of
following
a factor of 42. How many p
taken, there are k pictures left. What is t in terms of
admission
than
for
a
child’s
admission.
If
a
group
of
satisfy
the
equation
a
!
b
+
c
=
15
?
a factor of 42. How many possible values
in terms
taken, there are k pictures left. What is pt and
for are
p ? there
k ? of
4 adults
and
3
children
spent
$119
on
admission,
what
for p ?
(A)p and
2, 6,k3 ?
is the price of admission for one child? (A) One
(A) p ! k
(B)(A) 2, p7,!1 k
One
(B) Two
(A)(A)$11
(B) p " k
(B)
Two
(C) Three
(C)(B) 3, p5, "0 k
(B) $13
(C) k " p
(C)
Three
(D) Four
(C)
$16
(D)(C) 4,k3,"3 p
(D)$17
Four
(E) Five
(D) pk
(D)
(E) 5, 2, 4
(E)$18
Five
(D) pk
(E)
p
(E)
p
k
(E)
k
-26-
-26- 22
-
SECTION 6
Time — 25 minutes
18 Questions
SECTION 6
Time — 25 minutes
18 Questions
Turnanswer
to Section
6 to
(page
6) ofthe
your
answer sheet
answer the questio
Turn to Section 6 (page 6) of your
sheet
answer
questions
in thistosection.
3. The figure above shows four apartments in a building.
Directions:
This section
contains
two types
of questions.
You have
minutes1-8,
to complete
Directions:
This section
contains istwo
types
ofbyquestions.
You have
25 minutes
to complete
both types.
For 25
questions
solve both
In this building,
each apartment
occupied
only
each
problem
and
decide
which
is
the
best
of
the
choices
given.
Fill
in
the
corresponding
each
and decide
whichtoisSam,
the best
thelives
choices given. Fill in the corresponding circle on the answer sheet. You may circle o
oneproblem
person. Alice
lives next
and of
Paul
use
any
available
space for scratchwork.
use
anytoavailable
space
next
Alice and
Dara.forInscratchwork.
which apartment could Alice
5. The graph above shows various temperatures from 10
live?
A.M. to 6 P.M. of a given day. Which of the following
(A) 1 only
situations best fits the information on the graph?
(B) 2 only
(A) It rained a little, and then the Sun came out and
(C) 3 only
warmed things up.
(D) 2 or 3
(B)
The
mild temperature was lowered by a heavy
(E) 1 or 4
rain in the morning, and the temperature
dropped lower by evening.
(C) It was more windy in the morning than it was
in the evening, and the temperature was mild
throughout.
(D) The morning was cold, but the Sun later came
out and raised the temperature.
(E) The temperature decreased at a constant rate from
10 A.M. to 6 P.M.
1. A new roll of film has p pictures. After t pictures are
2. The prime number p is
A new
rollratio
of film
hasradius
p pictures.
After tot the
pictures are
2. The prime number p is a factor of 30 and is also
4. 1.What
is the
of the
r of a circle
a factor of 42. How m
taken, there are k pictures left. What is t in terms of
circumference
of the
circle? left. What is pt and
42.
How
many
possible
values
a
factor
of
k pictures
in terms
of
taken, there are
for are
p ? there
k?
for
p
?
p
and
k
?
(A) 1:2π
(A) One
(A) p ! k
(B)(A)1: πp ! k
(A) One
(B) Two
(B) p " k
(C) 1: πr
(B) Two
(C) Three
(B) p " k
(C) k " p
(C) Three
(D) π :1
(D) Four
(D) Four
(E) Five
(E)(C)2π k:1" p
(D) pk
(E) Five
(D) pk
p
(E)
p
k
(E)
k
-26-
- 23-26-
6. Rita’s dog weighed 5 pounds when she bought it.
7. If all four interior angles of
quadrilateral6 P have the
SECTION
SECTION
6
Over the next several years, the dog’s weight increased
same measure, which of the following statements must
Time — 25 minutes
Time — 25 minutes
by 10 percent per year. Which of the following
be true?
18 Questions
18 Questions
functions gives the weight, w, in pounds,
I. All sides of P have equal length.
of the dog after n years of weight gain at this rate?
II.
The
of
P are perpendicular.
Turnanswer
to Section
6 to
(page
6) diagonals
ofthe
your
answer
sheet
answer the questions
Turn to Section 6 (page 6) of your
sheet
answer
questions
in thistosection.
III. The measure of each interior angle of P is 90°.
(A) w n
5 0.1n
n
None
Directions:
This section
contains
two
types
of questions.
You have
minutes1-8,
to complete
(B) w n This
5 0.1
Directions:
section contains two types
of questions.
You have
25(A)
minutes
to complete
both types.
For 25
questions
solve both type
(B)
I
only
each
problem
and
decide
which
is
the
best
of
the
choices
given.
Fill
in
the
corresponding
each problem and decide
which is the best of the choices given. Fill in the corresponding circle on the answer sheet. You may circle on the
5 0.9 n
(C) w n
(C) II only
use any available space for scratchwork.use any available space for scratchwork.
(D) III only
n
(D) w n
5 1.1
(E) I, II, and III
(E) w n
5 n
1.1
x
in terms of z ?
5
1. A new roll of film has p pictures. After t pictures are
2. The prime number p is a fac
1. A new roll of film has p pictures. After t pictures are
2.(A)Thez prime number p is a factor of 30 and is also
a factor of 42. How many p
taken, there are k pictures left. What is t in terms of
z 1 of 42. How many possible values
(B)a factor
in terms
of
taken, there are k pictures left. What is pt and
for are
p ? there
k?
p and k ?
(C)for5 zp ?
(A) One
(A) p ! k
1
(D)(A)5 z One
(B) Two
(A) p ! k
(B) p " k
z Two
1
(B)
(C) Three
(E) 5
(B) p " k
(C) k " p
(C) Three
(D) Four
(C) k " p
(D) Four
(E) Five
(D) pk
(E) Five
(D) pk
p
(E)
p
k
(E)
k
8. If x
5 y and y
z
1, what is
-26-
- 24-26-
SECTION 6
Time — 25 minutes
18 Questions
SECTION 6
Time — 25 minutes
18 Questions
Turnanswer
to Section
6 to
(page
6) ofthe
your
answer sheet
answer the questi
Turn to Section 6 (page 6) of your
sheet
answer
questions
in thistosection.
Directions:
This section
contains
two types
of questions.
You have
minutes1-8,
to complete
Directions: This section contains two types
of questions.
You have
25 minutes
to complete
both types.
For 25
questions
solve both
each
andgiven.
decideFill
which
is the
best of the choices
given.
Fill in sheet.
the corresponding
each problem and decide which is the best
ofproblem
the choices
in the
corresponding
circle on
the answer
You may circle o
use any available space for scratchwork.use any available space for scratchwork.
1. A new roll of film has p pictures. After t pictures are
2. The prime number p is
1. A new roll of film has p pictures. After t pictures are
2. The prime number p is a factor of 30 and is also
a factor of 42. How m
taken, there are k pictures left. What is t in terms of
a factor of 42. How many possible values
in terms
taken, there are k pictures left. What is pt and
for are
p ? there
k ? of
for p ?
p and k ?
(A) One
(A) p ! k
(A) One
(B) Two
(A) p ! k
(B) p " k
(B) Two
(C) Three
(B) p " k
(C) k " p
(C) Three
(D) Four
(C) k " p
(D) Four
(E) Five
(D) pk
(E) Five
(D) pk
p
(E)
10. If 2 x + 3 y = 21 and x and y are positive integers,
p
k
(E)
what is one possible value of x ?
k
9. A snack machine has buttons arranged as shown above.
If a selection is made by choosing a letter followed by
a one-digit number, what is the greatest number of
different selections that could be made?
-26-
- 25-26-
11. A rectangular-shaped field has a perimeter of 400 feet
13. If x divided by one-half is
50, what is the
SECTION
6 value of x ?
SECTION
6
and a width of 80 feet. What is the area of the field in
Time — 25 minutes
Time — 25 minutes
square feet?
18 Questions
18 Questions
Turnanswer
to Section
6 to
(page
6) ofthe
your
answer sheet
answer the questions
Turn to Section 6 (page 6) of your
sheet
answer
questions
in thistosection.
Directions:
This section
contains
two types
of questions.
You have
minutes1-8,
to complete
Directions: This section contains two types
of questions.
You have
25 minutes
to complete
both types.
For 25
questions
solve both type
each
andgiven.
decideFill
which
is the
best of the choices
given.
Fill in sheet.
the corresponding
each problem and decide which is the best
ofproblem
the choices
in the
corresponding
circle on
the answer
You may circle on the
use any available space for scratchwork.use any available space for scratchwork.
12. If 3 × 10
n
+ 5 × 10
4
4
= 5.03 × 10 , what is the
value of n ?
1. A new roll of film has p pictures. After t pictures are
2. The prime number p is a fac
1. A new roll of film has p pictures. After t pictures are
2. The prime number p is a factor of 30 and is also
a factor of 42. How many p
taken, there are k pictures left. What is t in terms of
a factor of 42. How many possible values
in terms
taken, there are k pictures left. What is pt and
for are
p ? there
k ? of
for p ?
p and k ?
(A)? One
(A) p ! k
14. In ! ABC above, what is the length of AD
(A) One
(B) Two
(A) p ! k
(B) p " k
(B) Two
(C) Three
(B) p " k
(C) k " p
(C) Three
(D) Four
(C) k " p
(D) Four
(E) Five
(D) pk
(E) Five
(D) pk
p
(E)
p
k
(E)
k
-26-
- 26-26-
15. The sum of the positive odd integers less than 100 is
17. Fifty percent of the songs played on a certain radio
SECTION 6
subtracted from the sum of the positive even integers SECTION 6
station are 3 minutes long, 30 percent are 5 minutes
Time — 25 minutes
Time
—
25
minutes
less than or equal to 100. What is the resulting
long, and 20 percent are 2 minutes long. What is the
Questions
18 Questionsaverage (arithmetic mean) 18
difference?
number of minutes per song
played on this radio station?
Turnanswer
to Section
6 to
(page
6) ofthe
your
answer sheet
answer the questio
Turn to Section 6 (page 6) of your
sheet
answer
questions
in thistosection.
Directions:
This section
contains
two types
of questions.
You have
minutes1-8,
to complete
Directions: This section contains two types
of questions.
You have
25 minutes
to complete
both types.
For 25
questions
solve both
each
andgiven.
decideFill
which
is the
best of the choices
given.
Fill in sheet.
the corresponding
each problem and decide which is the best
ofproblem
the choices
in the
corresponding
circle on
the answer
You may circle o
use any available space for scratchwork.use any available space for scratchwork.
f x
kg x
2
16. The function f above is defined in terms of another
18. A large solid cube is assembled by gluing together
identical unpainted small cubic blocks. All six faces
function g for all values of x, where k is a constant.
of the large cube are then painted red. If exactly 27 of
30 and g t
8,
If t is a number for which f t
the small cubic blocks that make up the large cube have
what does k equal?
no red paint
ont them,
howare
many small2.cubic
1. A new roll of film has p pictures.
After
pictures
Theblocks
prime number p is
the large
cube?
1. A new roll of film has p pictures. After t pictures are
p is a factor of 30 and is also
2.make
Theup
prime
number
a factor of 42. How m
taken, there are k pictures left. What is t in terms of
a factor of 42. How many possible values
in terms
taken, there are k pictures left. What is pt and
for are
p ? there
k ? of
for p ?
p and k ?
(A) One
(A) p ! k
(A) One
(B) Two
(A) p ! k
(B) p " k
(B) Two
(C) Three
(B) p " k
(C) k " p
(C) Three
(D) Four
(C) k " p
(D) Four
(E) Five
(D) pk
(E) Five
(D) pk
p
(E)
p
k
(E)
k
STOP
If you finish before time is called, you may check your work on this section only.
Do not turn to any other section in the test.
-26-
- 27-26-
20. The words “dodging and shrinking” (line 34) primarily
SECTION 23.
7 The phrase “horrible immensities” (line 54) primarily
y, was
2 y + 7, y + 6, . . .
suggest that the narrator
Time — 25 minutesindicates
35 Questions (A) exaggerated information
(A) somewhat bothered by the children in the
13. audience
In the increasing sequence above, the first term is y
(B) unforeseen events
and the
difference between
any two consecutive
(B)
initially
overwhelmed
by the6)information
being terms
(C) historical
monstrositiesin this section.
Turn
to
Section
7
(page
of
your
answer
sheet
to
answer
the questions
is 3. What is the value of the fourth term in the
presented
(D) controversial debates
sequence?
(C) unable
to admit
to some
troubling
about
(E) incomprehensible
For each
question
in this
section,feelings
select the
best answer from among
the choices givenrealities
and fill in the corresponding
Directions:
(A) − 4sheet.
circle on theastronomy
answer
(D) refusing
24. The narrator suggests that the “echo-chamber effects,
(B) to
2 acknowledge the implications of space
travel
the cannot
music, expect
the solemnity”
(linesfriends
59-60)badly
are evidence
3.
You
to treat your
and no
(C)
5 test correctness and effectiveness
The following
sentences
(E) unwilling
to believe the studies being discussed
thatnotices.
one
of expression.(D)
Part 13
of each sentence or the entire sentence
(A)and
most
have feelings of great appreciation of
(A)
no adults
one notices
is21.
underlined;
beneath
each sentence
are five
of
19(“Moonless
In lines (E)
40-43
. . . Sun”),
theways
narrator’s
the
universe
(B)
and
have
no
one
notice
phrasing
the
underlined
material.
Choice
A
repeats
the
comment about the “arrangement” demonstrates
(B) without
most adults
would
rather not attend planetarium
(C)
notice
by
someone
original
phrasing;
the
other
four
choices
are
different.
If
a preference for
showsnotice by no one
(D) without
you think the original phrasing produces a better sentence
(A)ofirony
(C) without
contemporary
scientists
have an
inflated view
(E)
the result
of somebody
noticing
than any
the alternatives, select choice A; if not, select
(B)
inventiveness
of
the
importance
of
their
work
one of the other choices.
(C) symmetry
(D)memoirs
the show’s
promoters
do notbegin
fully with
appreciate
4. The
of President
Clinton
his the
In making
your
selection, follow the requirements of
(D)
ornamentation
true
nature
of
the
universe
childhood
in
Arkansas
and
culminate
in
his
rise
to
er than 2 ?standard written English; that is, pay attention to grammar,
(E) ambiguity
(E)presidency.
the show’s promoters understand that children are
the
choice of words, sentence construction, and punctuation.
entranced by special effects
(A) begin with his childhood in Arkansas and
Your
resultnearly
in the means
most effective
22. selection
In line 53,should
“fix” most
culminate
sentence—clear and precise, without awkwardness or
(A) focus
(B) that begin with his childhood in Arkansas and
ambiguity.
(B) prepare
culminate
(C) repair
(C) have begun with his childhood in Arkansas and
EXAMPLE:
(D) decide
culminate
Laura
Wilder published her first book
(E) Ingalls
influence
(D) have begun with his childhood in Arkansas and
and she was sixty-five years old then.
culminating
14. Let the function f be defined by f x
2 x 1.
(E) began with his childhood in Arkansas and are
(A) and she was sixty-five years old then
culminated
(B) whenIfshe1 was
f t sixty-five
10, what is the value of t ?
2
(C) at age sixty-five
years old
(D) upon the reaching of sixty-five years
5. Because of ignoring its potential, biofeedback is a
(A) 9.5
(E) at the time when she was sixty-five
medical therapy most physicians reject.
(B) 3
(A) Because of ignoring its potential, biofeedback is
(C) 3
a medical therapy most physicians reject.
(D) 9.5
bers
1. William Faulkner, being that he was a Southern writer,
(B) Biofeedback is rejected by most physicians
(E) 10.5
used Mississippi as a setting for most of his novels.
because of their ignoring its potential.
(C) Most physicians, because of ignoring the potential
(A) being that he was a Southern writer
of biofeedback, and rejecting it.
(B) a Southern writer
(D) Most physicians reject biofeedback because they
(C) while a writer from the South
ignore its potential.
(D) in origin a writer of the South
(E) A medical therapy rejected by most physicians,
(E) because of him being a writer from the South
caused
byon
ignoring
its potential,
If you finish before time is called, you may check your
work
this section
only.is biofeedback.
Do small
not turn to any other section in the test.
2. Tadpoles hatch and metamorphose into
replicas of adult frogs although remaining in
their aquatic birthplace.
ence
STOP
(A)
(B)
(C)
(D)
(E)
although remaining
while remaining
in spite of it remaining
due to their remaining
in the course of which they remain
- 28 -
6.20.InThe
Death
of a“dodging
Salesmanand
Willy
Loman mistakenly
words
shrinking”
(line 34) primarily
The phrase
(line
54)taxes
primarily
9.23.Although
the “horrible
candidateimmensities”
promised both
to cut
and
y, was
2noy flaws,
+ 7, ybelieving
+ 6, . . . which
suggestthat
thathis
thesons
narrator
indicates
believes
have
improve
services, he failed to keep either of them after
the election.
leads to many problems for the entire family.
(A) somewhat bothered by the children in the
(A) exaggerated information
13. audience
In the increasing sequence above, the first term is y
(B)Although
unforeseen
(A)
the events
candidate promised both to cut taxes
(A) believing which leads
and the
difference between
any two consecutive
(B)a belief
initially
overwhelmed
by the information
being terms
(C) and
historical
monstrosities
improve
services, he
(B)
that
leads
is 3. What is the value of the fourth term in the
(D)The
controversial
debatespromised both to cut taxes
(B)
candidate, having
(C) andpresented
which
is
to
lead
sequence? to some troubling feelings about
(C)the
unable
(E) and
incomprehensible
realities
improve services,
(D)
belieftoofadmit
which leads
(C) Although the candidate made promises both to cut
(A) − 4 this leads
(E) his astronomy
believing
(D) refusing
taxes andsuggests
improvethat
services,
he
24. The narrator
the “echo-chamber
effects,
(B) to
2 acknowledge the implications of space
travel
(D)
Having
promised,
first,
to
cut
taxes
and,
second,
the
music,
the
solemnity”
(lines
59-60)
are
evidence
7. Clara Barton,
(C) an5American nurse, whose influence as a
(E) unwilling
to believe
thecare
studies
being
discussed
that to improve services, the candidate
reformer
in the13
field
of health
almost
equals
that
(D)
(E) The candidate’s promises were both to cut taxes
of Florence
Nightingale.
(A) most adults have feelings of great appreciation of
and improve services, he
21. In lines (E)
40-4319(“Moonless . . . Sun”), the narrator’s
the universe
(A)
Clara Barton,
an“arrangement”
American nurse,
whose influence
comment
about the
demonstrates
(B)
most
adults would rather not attend planetarium
as a reformer
a preference
for
10. The students
showsfound fieldwork in the state forest more
(B) Clara Barton, who was an American nurse and
exciting
and dangerous
than any
of them
had view
(A) whose
irony influence as a reformer
(C) contemporary
scientists
have
an inflated
,
anticipated
having
to
be
rescued
by
helicopter
during
(B)An
inventiveness
of the importance of their work
(C)
American with influence as a nursing
a
fire.
(C) reformer,
symmetryClara Barton
(D) the show’s promoters do not fully appreciate the
(D)
ornamentation
true nature
of thetouniverse
(A)
anticipated,
having
be
(D) Clara Barton was an American nurse whose
greater than 2 ?
(E) influence
ambiguityas a reformer
(E) anticipated;
the show’swhen
promoters
understand
(B)
they had
to be that children are
entranced
by
special
effects
(C) anticipated: they had to be
(E) An American, Clara Barton who was a nursing
22. In line
53, “fix”
most
nearly
means
(D) anticipated: among which was their
reformer
and
whose
influence
(E) anticipated, and so they had been
(A) focus
8. During
the summer months, several thousand people
(B) prepare
11. Chinese watercolors have become more popular than
a day
the park, which is known for its waterfalls
(C) visit
repair
American and European artists who are their
and
formations.
(D)rock
decide
contemporaries.
(E) influence
(A) During the summer months, several thousand
(A) American and European artists who are their
people
a day
visit the
park,
whichbyis fknown
14.
Let the
function
f be
defined
x
2 x 1.
contemporaries
for its waterfalls and rock formations.
1
(B) contemporary American and European artists
(B) Known
rock
formations,
f its
t waterfalls
is the
value
of t ?
10, whatand
If for
2 thousand people a day visit the park
(C) those by contemporary American and European
several
artists of the period
during
summer months.
(A) the9.5
(D)
those
of American and European pictures of the
(C) Several
people a day visit the park
(B) thousand
3
same
period
during the summer months known for its
(C) 3
(E) those by contemporary American and European
waterfalls and rock formations.
(D) thousand
9.5
artists
numbers
(D) Several
people had visited the park
(E)
10.5
a day, which is known for its waterfalls and
rock formations during the summer months.
(E) During the summer months, knowing its
waterfalls and rock formations, several thousand
people a day visit the park.
umference
STOP
If you finish before time is called, you may check your work on this section only.
Do not turn to any other section in the test.
- 29 -
20. The words “dodging and shrinking” (line 34) primarily
23. The phrase “horrible immensities” (line 54) primarily
2 y ability
+ 7, y + 6, . . .
suggest that
the narrator
was
indicates
15. Both
Dorothy Sayers plus Carolyn Heilbrun
The following
sentences
testy,your
to recognize
grammar
and
usage
errors.
Each
sentence
contains
either
(A) somewhat bothered by the children in the
(A) exaggerated information
A
13. audience
In
above,
the first
term is y
a single error
orthe
no increasing
error at all.sequence
No sentence
contains
more
(B) written
unforeseen
events
have
scholarly works as well as popular
and
the
difference
between
any
two consecutive
than one
The
error,
if there
is one,
is underlined
(B) error.
initially
overwhelmed
by
the information
being terms
(C) historical monstrosities
is the
3. What
is thecontains
value ofanthe
fourth
termthe
in the
C
and lettered.presented
If
sentence
error,
select
(D) B
controversial debates
sequence?
one underlined
part
that
must
be
changed
to
make
the
(C) unable to admit to some troubling feelings about
(E) incomprehensible
realities a diversity of
murder
mysteries, demonstrating
sentence correct.
astronomy
4 sentence is correct, select choice E.
(A) If−the
D
In choosing
answers,
follow the requirements of standard
(D) refusing
24. The narrator suggests that the “echo-chamber effects,
(B) to
2 acknowledge the implications of space
No
error
talents
and interests.
written English.
travel
the music,
the solemnity” (lines 59-60) are evidence
(C) 5
(E) unwilling to believe the studies being discussed
that
E
(D) 13
EXAMPLE:
(A) most adults have feelings of great appreciation of
21. In lines (E)
40-4319(“Moonless . . . Sun”), the narrator’s
Although
16.
coach had predicted that the team
thethe
universe
delegates
and him immediately
The
other about
comment
the “arrangement”
demonstrates
(B)A most adults would rather not
B attend planetarium
a preference
for
A
B
C
shows
were surprised
would
have a winning
season, have
the fans
accepted
the resolution drafted by the
(A) irony
(C) contemporary
scientists
an inflated
view
D
(B) inventiveness
of the importance of their workC
neutral
states. No error
(C) symmetry
(D)
show’s
not fully appreciate
success
of promoters
by
thethe
the young,doinexperienced
players.the
(D)
ornamentation
true
nature
of
the
universe
E
er than 2 ?
D
(E) ambiguity
(E) the show’s
promoters understand that children are
No errorentranced by special effects
22. In line 53, “fix” most nearly means
E
ence
bers
(A)next
focus
12. By
year the old vaudeville theater had been
(B) prepare
A
(C) repair
converted
into two small theaters in which films
(D) decide
(E) influence
B
C
17. Because they must compete with a large chain of
A
super-stores that can afford to charge very low rates for
can be
error f be defined by f x
14.shown
Let the. No
function
2 x 1.
D1
E
10, what is the value of t ?
If f t
2
studying the writings of Emily Brontë
13. Some scholars
(A) 9.5
(B) 3 A
(C) increasingly
3
have become
interested in the
(D)
9.5
B
C
(E)
10.5
relationships between her siblings and she . No error
D
certain items, the owners of small hardware stores
know that you are unlikely to make much profit
B
C
and may , in fact, go bankrupt. No error
E
D
18. Although science offers the hope of preventing
A
E
serious genetic diseases, there is difficult ethical
B
If
you
finish
before
time
is
called,
you
may
check
your
work
on
this
section
only.
raised
by
possibility
of altering human
questions
the
Morales, who were swimmers on the United States
Do not turn to any other section in the test.
D
C
A
B
heredity. No error
team, set world records. No error
14. At the 1984 Olympic Games, John Moffet and Pablo
STOP
C
D
E
E
- 30 -
20. The words “dodging and shrinking” (line 34) primarily
y, was
2careers
y + 7,inybusiness
+ 6, . . . and law
suggest
that the narrator
19. In
the nineteenth
century,
23. The phrase “horrible immensities” (line 54) primarily
indicates , Whistler’s paintings, unlike Klee , are
23. Obviously
B
(A) somewhat bothered by the children in the
(A) Aexaggerated information
13. audience
In the increasing sequence above, the first term is y
(B)
unforeseen
events
but it between
were prestigious,
did not require
conventional in their subject matter. No error
and the
difference
any twopractitioners
consecutive
(B)
overwhelmed
by theCinformation
being terms
(C) historical monstrosities
A initially
B
is 3. What is the value of the fourth term in the
Ddebates
C
E
presented
(D) controversial
sequence?
college
degrees.
to
No error
(C)hold
unable
to admit
to some
troubling feelings about
(E) incomprehensible realities
D astronomy
E
(A) − 4
24. A newly formed organization of homeowners and
(D) refusing
24. The narrator suggests that the “echo-chamber effects,
(B) to
2 acknowledge the implications of space
travel
the music, the solemnity” (lines 59-60) are evidence
(C)
5
20. Chess
players find
that playing
againstbeing
a computer
business
people have met with the transportation
(E) unwilling
to believe
the studies
discussed
that
(D) 13
A
(A) most adults have feelings of great appreciation of
19(“Moonless . . . Sun”), the narrator’s
21. isInhelpful
lines (E)
40-43
to improve their skills, even though no
to voice its concerns about plans for
department
the universe
comment about the “arrangement” demonstrates
(B)
most
adults
Bwould rather not
C attend planetarium
A
D
B
C
a preference for
shows
chess-playing computer has yet won a championship.
Noinflated
error view
a(C)
shopping
mall in the
community.
(A) irony
contemporary
scientists
have an
D
(B) inventiveness
of the importance of their work E
(C)error
symmetry
(D) the show’s promoters do not fully appreciate the
No
(D)
ornamentation
true nature of the universe
greater than 2 ?
25. Although she considers her chemistry research
(E)E ambiguity
(E) the show’s promoters understand that children are
A entranced by special effects
21.
of twelve
22. Reaching
In line 53, lengths
“fix” most
nearlyinches,
means banana slugs are
complete , she has heeded her professor’s advice
B
A
B
C
(A) focus
(B)much
prepare
the
larger of all the slug species that inhabit
and is conducting three additional experiments.
(C) repair
C
D
D
(D) decide
North
America.
No
error
No error
(E) influence
E
E
14. Let the function f be defined by f x
2 x 1.
umference
numbers
1
t
whather
is the
value of t ?
10, wore
If f proudly
sister’s
22. Maya Madera
2
A
(A) 9.5
most popular invention, a watch for winter campers
(B) 3
B
(C) 3
that flashes
temperature in lighted numerals and
(D) the9.5
C
(E) 10.5
26. Princeton University officials first broke with
A
a tradition of awarding honorary degrees only
B
to men when they awarded it to author Willa
C
D
sends out a loud alarm when pressing a button .
D
No error
D
E
Cather. No error
E
STOP
If you finish before time is called, you may check your work on this section only.
Do not turn to any other section in the test.
- 31 -
rence
20. The words “dodging and shrinking” (line 34) primarily
27. His love of politics led yhim
2 y to
+ volunteer
7, y + 6, in
. . .local
suggest that the narrator, was
A B
(A) somewhat bothered by the children in the
13.
In
the
increasing
sequence
above, theoffice
first term is y
in a government
campaigns
as well as a job
audience
and
the
difference
between
any
two
consecutive
D by the information being terms
(B) initiallyCoverwhelmed
is
3.
What
is
the
value of the fourth term in the
presented
in the state
capital. No error
sequence?
(C) unable to admit to some troubling feelings about
E
astronomy
(A) − 4
(D) refusing
to
acknowledge
the implications of space
(B) 2
travel
28. Now that(C)
Michiko
5 finished the research, she feels
(E) unwilling to believe
A the studies being discussed
(D) 13
19(“Moonless
about. writing
confident
21. reasonably
In lines (E)
40-43
. . Sun”),her
thepaper
narrator’s
C
commentBabout the “arrangement”
demonstrates
a preference for
on the rise of the progressive movement in the
(A) irony
D
(B) inventiveness
United
States. No error
(C) symmetry
(D) ornamentation
E
er than 2 ?
(E) ambiguity
bers
29. The condition known as laryngitis usually causes
22. In line 53, “fix” most nearly means
A
(A) focus
the
cords and surrounding tissue to swell, thus
(B)vocal
prepare
B
(C) repair
(D) decide
preventing
the cords to move freely . No error
(E) influence
C
D
E
14. Let the function f be defined by f x
1
If f t
2
2x
1.
10, what is the value of t ?
(A) 9.5
(B) 3
(C) 3
(D) 9.5
(E) 10.5
23. The phrase
primarily
Directions:
The “horrible
followingimmensities”
passage is an(line
early54)
draft
of an
indicates
essay. Some parts of the passage need to be rewritten.
(A) passage
exaggerated
information
Read the
and select
the best answers for the
(B)
unforeseen
events
questions that follow. Some questions are about particular
(C) or
historical
sentences
parts of monstrosities
sentences and ask you to improve
(D)
controversial
debates
sentence structure or word
choice. Other questions ask you
(E)
incomprehensible
realities
to consider organization and development.
In choosing
answers, follow the requirements of standard written
24. The narrator suggests that the “echo-chamber effects,
English.
the music, the solemnity” (lines 59-60) are evidence
that
Questions
30-35 are based on the following.
(A) most adults have feelings of great appreciation of
the universe
(1)(B)
A significant
problem
across
stateplanetarium
is garbage.
most adults
wouldall
rather
notour
attend
(2) Our landfills
are
full.
(3)
It
seems
that
we
must
either
shows
find new
sites
for
landfills
or
employ
other
methods
of
(C) contemporary scientists have an inflated view
disposal, likeofincineration.
(4) Unfortunately,
the importance
of their work there are
drawbacks
to every
solution
that do
they
(5) Polluted
(D) the
show’s
promoters
notthink
fullyof.
appreciate
the
runoff watertrue
oftennature
results
from
landfills.
(6)
With
of the universe
incineration
ofshow’s
trash, you
get air pollution.
People
are are
(E) the
promoters
understand(7)
that
children
criticized forentranced
not wanting
to live near
a polluting waste
by special
effects
disposal facility, but really, can you blame them?
(8) Recycling can be an effective solution, but owners of
apartment complexes and other businesses complain that
recycling adds to their expenses. (9) Local governments
enjoy the benefits of taxes collected from business and
industry. (10) They tend to shy away from pressuring such
heavy contributors to recycle.
(11) Perhaps those of us being concerned should
encourage debate about what other levels of government
can do to solve the problems of waste disposal. (12) We
should make a particular effort to cut down on the
manufacture and use of things that will not decompose
quickly. (13) Certainly we should press individuals,
industries, and all levels of government to take responsible
action while we can still see green grass and trees between
the mountains of waste.
30. Which of the following would fit most logically
between sentences 1 and 2 ?
(A) A sentence citing examples of states that have
used up available landfills
(B) A sentence citing examples of successful
If you finish before time is called, you may check your
work ontothis
section only.
alternatives
landfills
Do not turn to any other section
in
the
test.
(C) A sentence citing the number of new landfills in
the state
(D) A sentence citing the number of illnesses blamed
on polluted water in the state
(E) A sentence citing the average amount of trash
disposed of annually by each person in the state
STOP
- 32 -
umference
34.
context,
of the
following is(line
the best
way to
23. InThe
phrasewhich
“horrible
immensities”
54) primarily
combine
indicatessentences 9 and 10 ?
31.
of the“dodging
following
is the
best way
to phrase
the
20. Which
The words
and
shrinking”
(line
34) primarily
y, was
2 y +47(reproduced
, y + 6, . . . below) ?
underlined
portion
of sentence
suggest that
the narrator
Unfortunately,
are drawbacks
to every
solution
(A) somewhatthere
bothered
by the children
in the
In
the of.
increasing sequence above, the first term is y
that 13.
theyaudience
think
and the
difference between
any two consecutive
(B) (as
initially
overwhelmed
by the information
being terms
(A)
it is
now)
is
3.
What
is
the
value
of
the
fourth
term
in the
presented
(B) thatsequence?
was thought of
(C) unable to admit to some troubling feelings about
(C) that they have previously come up with
astronomy
4 are proposals
(A) −there
(D) to which
(D) refusing
to
the implications of space
(B)
2 acknowledge
(E) thattravel
has been
proposed
(C) 5
(E) unwilling to believe the studies being discussed
13
32. Which of(D)
the following
is the best way to revise and
combine
sentences
5
and
6 (reproduced
below)
?
(E)
19
21. In lines 40-43 (“Moonless
. . . Sun”), the
narrator’s
comment
about
the
“arrangement”
demonstrates
Polluted runoff water often results from landfills. With
a preferenceoffor
incineration
trash, you get air pollution.
(A) With
ironylandfills, polluted runoff water will result,
(A)
(B) inventiveness
and whereas with incineration of trash, you get
(C) symmetry
air pollution.
(D) While
ornamentation
greater than 2 ?
(B)
on the one hand are landfills and polluted
(E) ambiguity
runoff water, on the other hand you have air
pollution in the case of incineration of trash.
22. In line 53, “fix” most nearly means
(C) Landfills often produce polluted runoff water, and
trash incineration creates air pollution.
(A) focus
(D)
(B) Landfills
prepare and incineration that produce water and
(C) repair
air pollution.
(D) Runoff
decide water is from new landfills; from
(E)
(E) influence
incineration of trash, there is air pollution.
(A)
governments
enjoy the benefits of taxes
(A) Local
exaggerated
information
collected from
business and industry, as they
(B) unforeseen
events
(C) historical
monstrosities
tend to shy
away from pressuring such heavy
(D) controversial
debates
contributors to
recycle.
(E) Because
incomprehensible
realities enjoy the benefits of
(B)
local governments
taxes collected from business and industry, they
24. The narrator
thepressuring
“echo-chamber
effects,
tend to suggests
shy awaythat
from
such heavy
the music,
the solemnity”
(lines 59-60) are evidence
contributors
to recycle.
that However, local governments enjoy the benefits of
(C)
taxesadults
collected
business
and appreciation
industry, they
(A) most
havefrom
feelings
of great
of
tend
to
shy
away
from
pressuring
such
heavy
the universe
contributors
to recycle.
(B) most
adults would
rather not attend planetarium
(D) In shows
addition to enjoying the benefits of taxes
collected fromscientists
business have
and industry,
local
(C) contemporary
an inflated
view
governments
tend toofshy
away
of the importance
their
workfrom pressuring
business
industrydo
into
(D) the
show’sand
promoters
notrecycling.
fully appreciate the
(E) Local
enjoying the benefits of taxes
truegovernments,
nature of the universe
collected
business
and industry,
they tendare
(E) the
show’sfrom
promoters
understand
that children
toentranced
shy awaybyfrom
pressure
to recycle.
special
effects
35. Which of the following is the best phrasing for
the underlined portion of sentence 11 (reproduced
below) ?
14. Let the function f be defined by f x
2 x 1.
33. If sentence 8 were rewritten to begin with the clause
1
can what
be aniseffective
solution,
“Although
f t
the value
of t ? ” the
10,
If recycling
next words 2would most logically be
numbers
9.5of apartment complexes and other
(A) and(A)
owners
(B) 3 complain
businesses
(B) yet (C)
owners3 of apartment complexes and other
businesses
(D) 9.5complain
(C) owners
apartment complexes and other
(E) of10.5
businesses complain
(D) mostly owners of apartment complexes and other
businesses are complaining
(E) owners of apartment complexes and other
business complained
Perhaps those of us being concerned should encourage
debate about what other levels of government can do to
solve the problems of waste disposal.
(A)
(B)
(C)
(D)
(E)
(as it is now)
those of us who are concerned
those concerned ones of us
we, being among those who are concerned,
we who are those being concerned
STOP
If you finish before time is called, you may check your work on this section only.
Do not turn to any other section in the test.
STOP
If you finish before time is called, you may check your work on this section only.
Do not turn to any other section in the test.
- 33 -
NO TEST MATERIAL ON THIS PAGE.
- 34 -
SECTION 8
Time — 20 minutes
19 Questions
SECTION 8
Time — 20 minutes
19 Questions
Turn
Section
(page 7) ofinyour
f your answer sheet
to to
answer
the8questions
this answer
section.sheet to answer the questions in this section.
Directions:
Foramong
each question
in this
section,
select
thecorresponding
best answer from among the choices given and fill in the corresponding
on, select the best
answer from
the choices
given
and fill
in the
circle on the answer sheet.
3. The
serious
purpose
the paper-airplane
flying as
3. Historian Carlo Botta often contradicted himself, as
Historian
Carlo
Bottaofoften
contradicted himself,
Each sentence below has one or two blanks, each blank
contest,
which
attracted many
novel
and sometimes
when he first championed and then ------- the ideals of
when
he
first
championed
and
then
------the
ideals of
indicating that something has been omitted. Beneath
truly
------- Revolution.
entries, was to determine whether any
the French Revolution.
the French
the sentence are five words or sets of words labeled A
------- aerodynamic designs could be discovered.
through E. Choose
the word or(B)
set of
words that, when
(A) invoked
investigated
(C) conceived
(A) invoked
(B) investigated
(C) conceived
(A)(D)unorthodox
conventional
inserted in the sentence,
best fits the
of the
(D) coveted
(E) meaning
denounced
coveted . .(E)
denounced
(B) bizarre . . revolutionary
sentence as a whole.
(C)
. . imaginative
4. Luisa worked with extreme precision, ------- that
4.
Luisaderivative
worked with
extreme precision, ------- that
Example:
(D)
mundane
. predictable
served her well in her law career.
served
her well. in
her law career.
(E) ungainly . . aesthetic
Hoping to ------- the dispute, negotiators proposed
proposed
(A) a meticulousness
(B) an effrontery
(A) a meticulousness
(B) an effrontery
a compromise that they felt would be ------- to both
--- to both
(C) an inhibition
(D) a litigiousness
(C)
an
inhibition
(D)
a litigiousness
4. Carson presents her case so strongly
and logically
labor and management.
(E) an impetuousness
(E)
an
impetuousness
that only the prejudiced or the ------- will attempt to
(A) enforce . . useful
------- her.
In 1916 Yellowstone National Park had only 25 bison, 5. In 1916 Yellowstone National Park had only 25 bison,
(B) end 5.
. . divisive
(A)
impartial
. . defy
but.the
population has since ------- to more than 2,000.
(C) overcome
. unattractive
but the
population
has since ------- to more than 2,000.
(B) doubtful . . champion
(D) extend . . satisfactory
(A) dispersed
(B) mediated
(C) attenuated
(A) dispersed
(B) mediated
(C) attenuated
(C)
gullible . . believe
(E) resolve . . acceptable
(D) burgeoned
(E) reconciled
burgeoned
(E) reconciled
(D)(D)obstinate
. . contradict
(E) irrational . . follow
6. Though
surgeon
and researcher
CharlestoDrew
an audiences, 1. Most
pioneers
------thisby
valley
on their
the never 6. Though surgeon and researcher Charles Drew never
Originally
------mainly
young,
urbanjourney
audiences,
enjoyed
celebrity,
he trulyfrequent
deserveslandslides
to be ------- for his
enjoyed
truly deserves opinions
to be ------ppreciative
West
because
rugged
terrain
rap music
wasits
ultimately
------- and
by its appreciative
5. Over
thecelebrity,
years the he
anthropologist’s
hadfor his
life’s
achievements.
life’s
achievements.
made
it aof
------place
for travelers.
listeners
all ages
across
the country.
-------: he refused to tolerate new ideas and nothing
could
change his mind.
(A)
mollified
(B)
lionized
(C)
accosted
(A) mollified
(B) lionized
(C) accosted
(A) flanked
admired. .. .fascinating
embraced
(D)
galvanized
(E)
vilified
(D)
galvanized
(E)
vilified
(B) avoided
. . necessary
performed
. . condemned
(A) digressed
(B) proliferated
(C) ossified
each blank
Beneath
labeled A
that, when
ng of the
(C) encompassed
. . curious
derided . . ignored
(D) enjoyed
. . troublesome
appropriated
. . relinquished
(E) skirted
. . hazardous
applauded
. . instigated
(D) germinated
6. As the first ------- of the political campaign, the
senator unleashed a spirited verbal attack on her
leading opponent.
n, the author of 2. Ballads
often
praisethat
popular
figures
who have
It was out
of ------Professor
Green,
the author of
eld, described
performed
featsrespected
that many
perceive
-------,
such as
several highly
books
in hisasfield,
described
defending
poor or resisting
------- authority.
himself to the
his colleagues
as -------.
(A)
(B)
(C)
(D)
(E)
(E) incubated
(A) salvo
(B) encore
(C) palliative
(D) concession
(E) demurral
embarrassment
. . a paragon
modest
. . acceptable
magnanimity. .. legitimate
. an avenger
inescapable
insolence . ..a. pedant
insufficient
overpowering
egotism . . .an
apprentice
admirable
. unjust
modesty . . a dilettante
unbelievable
. . tolerable
- 35 -
The two passages
below are followed by questions based
on their content
SECTION
8
SECTION
8 and on the relationship between the two passages.
Answer
the questions on the basis of what is stated or
implied
in minutes
the passages and in any introductory material that may be
Time —
20 minutes
Time
— 20
provided.
19 Questions
19 Questions
Byanswer
the time the
became popular,
Endowment had
are based
the following
passages.
ur answerQuestions
sheet
to7-19
answer
the8on
questions
this
section.sheet to
Turn
to
Section
(page
7) ofinyour
answer
theshow
questions
in thisthe
section.
vanished from the credits, its job done.
40
When you’re starting out, it seems like nobody wants
The
following
passages
are
taken
from
testimony
given
ect the best
answer from
the choices
given
and fill
in the
Directions:
Foramong
each question
in this
section,
select
thecorresponding
best answer from
among
choices
fillyou
in the
corresponding
to give
you the
a dime,
andgiven
then, and
when
have
big success
before congressional committees about how government
circle on the answer sheet.
and have everything you could ever want, people can’t do
funding affects the arts. The author of the first passage is
enough for you. The Endowment is there at the beginning,
a writer and radio entertainer; the author of the second
3.a Historian
Carlo
Botta often contradicted himself, as
3.andHistorian
Botta
often contradicted himself, as
that’s theCarlo
beauty
of it.
passage
is
novelist
and
critic.
blank Each sentence when
belowhehas
onechampioned
or two blanks,
first
and each
then blank
------- the ideals of
when he first championed and then ------- the ideals of
ath
indicating thatthe
something
has been omitted. Beneath
Passage
2 Revolution.
French Revolution.
the French
Passage
1 are five words or sets of words labeled A
ed A
the
sentence
I love
my country’s
in a pre(A) invoked
investigated
(C) conceived 45
when
(A)
invoked
(B) government
investigatedfor its
(C)attempt
conceived
through
Choose
thehave
wordhonored
or(B)
set of
wordswhen
that, they
when
All E.
governments
artists
are
carious
world
to
sustain
a
peaceful
order
in
which
work
can
(D) coveted
(E)
denounced
he
(D) coveted
(E) denounced
inserted
the sentence,
best fitsand
the almost
meaning
of the
old andinsaintly
and successful
dead,
but twentybe
done
and
happiness
can
be
pursued,
not
for
the
good
of
sentence
as aago
whole.
five years
Congress decided to boldly and blindly
state,worked
but in awith
stateextreme
that exists
for our good.
4. Luisa worked with extreme precision, ------- that
4.theLuisa
precision,
------- that
support
the
arts—support
the act of creation itself— and
Line Example:
I love my
government
not career.
least for the extent to which
served her well in her law career.
served
her
well
in
her
law
5 to encourage artists who are young and dangerous and
50 it leaves me alone. My personal ambition has been simply
sed
Hoping toand
------dispute,
unknown
very
much
alive.negotiators
This courageous
legislation
(A)
athe
meticulousness
(B) proposed
an effrontery
(A) by
a meticulousness
(B)This
an effrontery
to live
the work of my pen.
is not a very fastidious
both
a compromise
that an
they
felt would be
to both
has
changed American
life.
(C)
inhibition
(D)------a litigiousness
(C)
an
inhibition
(D)
a
litigiousness
ambition.
If
I
were
aware
of
large
amounts
of federal money
labor
andyears
management.
Forty
ago,
men or women meant to
(E)if American
an impetuousness
(E)to an
impetuousness
available
purveyors
of the written word, I would attempt
have
(A) artistic
enforcecareers,
. . usefulthey got on the train to New York.
to gain access to it and hope to please the administrators of
10 Today,
you
can
be a violinist
in North
Carolina,
writer
5.
In
1916
Yellowstone
National
Park ahad
onlyin25 bison,
1916asYellowstone
National
Park had
onlyand
25book
bison,
(B) end . . divisive
55 5.
thisInfund
I hope to please
magazine
editors
Iowa,
a painter
This is has
a small
lovely
revolubutin
population
sinceand
------to more
than 2,000. buyers.
but the population has since ------- to more than 2,000.
(C) overcome
.the
. Utah.
unattractive
tion
the National
Endowment for the Arts (NEA) has
(D) that
extend
. . satisfactory
But Idispersed
would rather(B)
have
as my patron
host of anony(A)
dispersed
(B) mediated
(C) thouattenuated
(A)
mediated
(C)a attenuated
helped
to bring
about.
The Endowment
has fostered
(E) resolve
. . (D)
acceptable
mous(D)
citizens
digging into
their
own pockets for the price
burgeoned
(E)
reconciled
burgeoned
(E)
reconciled
sands of artistic works—many of which will outlive you
of a book or a magazine than a small body of enlightened
15 and me—but even more important, the Endowment has
60 6.
andThough
responsible
people
public
funds.
I would
6.
Though
surgeon
and
researcher
Charles
Drew
never
surgeon
and administering
researcher Charles
Drew
never
how ------we think
aboutbythe
arts. Today,
no American
iences, 1.changed
Originally
mainly
young,
urban audiences,
rather
chance
my
personal
vision
of
the
truth
striking
celebrity,
truly deserves
enjoyed celebrity, he truly deserves to be ------- forhome
his
iative
family
can beenjoyed
secure
against------thehedanger
one to
of be
its ------chil- for his here
rap music
was
ultimately
by itsthat
appreciative
andachievements.
there in the chaos of publication that exists than
life’s
achievements.
life’s
dren
may
decide
to
become
an
artist.
listeners of all ages across the country.
attempt to filter it through a few sets of official, honorable,
I grew up(A)
in a mollified
family that never
attended concerts
or
(B) lionized
(C) accosted
mollified scrutinizers.
(B) lionized
(C) accosted
and(A)
public-spirited
(A)
admired
.
.
embraced
20 museums, never
bought
books. I never
imagined that a
(D)
galvanized
(E) vilified
(D)
galvanized
(E)
vilified
65
The
realms
of
scientific
research
are
now inextricably
(B) could
performed
. . condemned
person
be a writer.
involved with government funding. Can we fear that the
(C)
derided
.
.
ignored
Twice in my life, at crucial times, grants from the
humanities might become similarly dependent? If I try to
(D) appropriated
. . relinquished
Endowment
made it possible
for me to be a writer. The
think of who in the last century has most brilliantly illumi(E)
applauded
.
.
instigated
first, in 1969, arrived when I was young, broke, married
nated our sense of humanity, which I take to be the end
25 with a baby, living on very little cash and a big vegetable
70
purpose of the humanities, I think of Freud and Kafka, of
author of 2. It was out of ------- that Professor Green, the author of
garden. I was writing for The New Yorker at the time, but
Proust and Joyce, of Whitman, of Henry James. I wonder
escribed
several
highly
respected
books
in
his
field,
described
they weren’t aware of it. I wrote every morning and every
how many of these brave, strange, stubborn spirits would
himself
to
his
colleagues
as
-------.
night. I often had fantasies of finding a patron— a beggar
have wanted subsidies from their governments.
would
at my door,
give him an egg salad sand(A) appear
embarrassment
. . aI’d
paragon
How can public-salaried officials not think in terms of
30 wich,
suddenly he’d
turn
into a man in a pinstripe suit,
(B) and
magnanimity
. . an
avenger
75 respectability, of social optimism, of broad and uncontroPrince
from a. .philanthropic
foundation. But instead, I
(C) Bob
insolence
a pedant
versial appeal? How can legislators, asked to vote tax money
got(D)
a letter
offering
a job for one month in the Writers
egotism
. . anme
apprentice
away, not begin to think of guidelines that insidiously edge
in the
program
in Minneapolis, funded by the
(E) Schools
modesty
. . a dilettante
toward censorship?
NEA, which sent young writers into the schools to read
If government money becomes an increasingly impor35 and teach.
80 tant presence in the financing of the humanities, is there
In 1974 a grant from the NEA enabled me and my cola danger, I respectfully ask, of humanists becoming
leagues at a public radio station to start a new radio series.
politicians?
- 36 -
7. The argument in Passage 1 is supported primarily by
SECTION 8
12. Lines 40-43 (“When . . . for you”) suggest that the
SECTION 8 author of Passage 1 would agree with which of the
(A)minutes
a theory of how art is created
Time — 20
Time — 20 minutesfollowing observations?
(B)
the author’s personal experiences
19 Questions
19 Questions
(C) examples of renowned artists
(A) Those who never succeed value success most.
(D) evidence of the harmful effects of arts funding
(B) The love of money is the root of all evil.
f your answer sheet
to to
answer
the8toquestions
inyour
this
section.sheet to answer
Section
(page
answer
thesucceeds
questions
in this section.
(E)Turn
emotional
appeals
uphold 7)
theof
rights
of citizens
(C) Nothing
like success.
(D) Make a virtue of necessity.
Time
money.given and fill in the corresponding
8.answer
Which
isFor
aamong
likely
response
bythis
the section,
author
ofselect
Passage
2 answer from(E)
on, select the best
from
the choices
given
and fill
in the
Directions:
each question
in
thecorresponding
best
among
theischoices
of artists as “dangerous” (line 5) in
circletoonthe
thedescription
answer sheet.
Passage 1 ?
13. The author of Passage 2 would most likely criticize the
author of Carlo
Passage
1 onoften
the grounds
that himself, as
3. Historian
Carlorarely
Bottaprevent
often contradicted
3. Historian
Botta
contradicted
Practical
concerns
artists
from himself, as
, each blank Each(A)
sentence
below
has
one
or
two
blanks,
each
blank
when he
championed
and
then ------- the ideals of
when
he firstdetracts
championed
then ------- the
ideals
of
speaking
outfirst
on
controversial
issues.
(A) humor
from and
the seriousness
of the
issue
. Beneath
indicating that
something
has
been omitted.
Beneath
the
Frenchnot
Revolution.
the French
Revolution.
(B)
Artists
should
be
permitted
to
undermine
the
being
discussed
s labeled A
the sentence are five words or sets of words labeled A
values
society.
(B) invoked
public funding
often given to(C)
artists
who do not
(A) of
invoked
investigated
(C) conceived
s that, when
(A)
(B) is
investigated
conceived
through E. Choose
thetheir
word
or(B)
set of
words that, when
(C)
Artists
will
cease
taking
risks
if
they
come
to
need
it
(D) coveted
denounced
ng of the
(D) coveted
(E) denounced
inserted in the sentence,
best fits (E)
the meaning
of the
(C) it is invalid to assume that artists can also be
sentence as depend
a whole.on government money.
(D) The
futureworked
of the arts
inextreme
the United
States depends
teacherswith extreme precision, ------- that
4. Luisa
with
precision,
------- that
4. Luisa worked
Example: on served
whether
young
artists
can
continue
to
shock
(D)
taxes
will in
be her
too law
highcareer.
if the government supports
her well in her law career.
served her well
the
public.
the
arts
proposed
Hoping to ------dispute, negotiators
proposed
(A) athe
meticulousness
(B)how
an to
effrontery
(A)
meticulousness
(B)
effrontery
(E) Experienced
artists
know
better
excite the
(E) apublic
funding is just
as an
likely
to inhibit artists as
---- to both
a compromise
that
they
felt
would
be
------to both
(C)
ando
inhibition
(D) a litigiousness
(C) an
inhibition
(D)
a litigiousness
public
than
young
artists.
it
is
to
encourage
them
labor and management.
(E) an impetuousness
(E) an impetuousness
. . useful
9.(A)In enforce
lines 10-11,
Passage 1, the author refers to North
14. In line 57, Passage 2, “host” most nearly means
In
1916
Yellowstone
(B)Carolina,
end 5.
. . divisive
Iowa,
and
Utah to National Park had only 25 bison, 5. In 1916 Yellowstone National Park had only 25 bison,
(A)thelarge
numberhas since ------- to more than 2,000.
but .the
population has since ------- to more than 2,000.
but
population
(C) overcome
. unattractive
(A)
prove
that
certain
environments
support
creativity
(B)
sustaining
organism
(D) extend (A)
. . satisfactory
dispersed
(B) mediated
(C) attenuated
(A)
dispersed
(B) mediated
(C) attenuated
better
than
others
(C)
provider
(E) resolve . . (D)
acceptable
(E) reconciled
(D)proprietor
burgeoned
(E) reconciled
(B) support the burgeoned
argument by mentioning
how few
(D)
exceptions there are to it
(E) sponsor
6. Though
surgeon andforresearcher
Charles
(C) show
that opportunities
artistic
areDrew
now never 6. Though surgeon and researcher Charles Drew never
an audiences, 1. Originally
------mainly by young,
urbansuccess
audiences,
enjoyed
celebrity, he truly deserves to be ------- for his 15. enjoyed
celebrity,
he truly
be ------for his
widely
The question
in lines
66-67deserves
suggeststothat
the author
appreciative
rap music
was available
ultimately ------- by its appreciative
life’s
achievements.
life’s
achievements.
(D)
suggest
that
legislators
should
listen
to
their
of
Passage
2
believes
that
federal
funding
of
scientific
listeners of all ages across the country.
constituents
research
(A) mollified
(B) lionized
(C) accosted
(A)
mollified
(B) lionized
(C) accosted
(A)
. . embraced
(E) admired
offer a (D)
parallel
between arts(E)
funding
and democracy
galvanized
vilified
(D)
galvanized
(E)
vilified
(A)
encourages
deceit
(B) performed . . condemned
(B) undermines autonomy
. . in
ignored
10. (C)
The derided
statement
lines 16-18, Passage 1 (“Today . . .
(C) encourages an unhealthy competitiveness
(D)
appropriated
.
.
relinquished
artist”) is best described as an example of
(D) develops a superficial sense of loyalty
(E) applauded . . instigated
(A) an ironic comment
(E) spends public money under false pretenses
(B)
an
emotional
plea
n, the author of 2. It was out of ------- that Professor Green, the author of
(C) a moral
16. The author of Passage 2 most likely thinks that the
eld, described
several
highlypronouncement
respected books in his field, described
(D) a definition
of a keyasconcept
individuals named in lines 70 -71 would have
himself
to his colleagues
-------.
(E) a generalization supported by research
(A) supported the idea of providing artists with
(A) embarrassment . . a paragon
government funding
(B)
magnanimity
.
.
an
avenger
11. The “man in a pinstripe suit” (line 30, Passage 1) is
(B) avoided writing about controversial topics if
(C) insolence . . a pedant
(A) egotism
a fastidious
doing so brought them more funding
(D)
. . anbureaucrat
apprentice
(B)
a
character
in
a
novel
(C)
been
even more influential if they had received
(E) modesty . . a dilettante
(C) a famous writer
government funding
(D) an anonymous critic
(D) embraced the romantic image of the starving
(E) an imagined benefactor
artist
(E) refused to submit their creativity to outside
control
- 37 -
17. The final sentence of Passage 2 serves to
SECTION 8
19. Which of the following situations would support the
SECTION 8 position taken in Passage 1 and provide examples
emphasize the moral dilemmas that artists face
Time — 20(A)
minutes
Time — 20 minutes contrary to the argument in Passage 2 ?
19 Questions when selling their work
19 Questions
(B) indicate why artists are so often in need of finanI. A federally sponsored photographer displayed an
cial support
innovative collection of photographs that offended
ur answer sheet
to
answer
the
this
section.
Turn
to
Section
8questions
(page
7)not
ofinhave
your
the
thissegment
section.
(C)
suggest
that the
public
should
to answer
subsidize sheet to answer
both
thequestions
sponsor and ainlarge
of the public.
the art preferred by bureaucrats
II. The most original works of a certain brilliant
(D) from
warn
of
thequestion
likelihood
artistic
were
those
by kings.
ect the best
answer
the
choices
given
andcompromise
fill
in the
Directions:
Foramong
each
in of
this
section,
select
thecorresponding
best answer from amongcomposer
the choices
given
andcommissioned
fill in the corresponding
(E)thelink
arts funding
III. A theatrical troupe from Harlem achieved promicircle on
answer
sheet. in the United States with other
social programs
nence by drawing large audiences from its local
community.
3. Historian Carlo Botta often contradicted himself, as
3. Historian
Carlo Botta often contradicted himself, as
blank Each
below
has
one
or
two
blanks,
each
blank
18. sentence
Which of
the
following
is
an
assumption
in
Passage
2
when he first championed and then ------- the ideals of
when
he
first championed and then ------- the ideals of
(A)
I
only
ath
indicating
thatauthor
something
has been
omitted.
that the
of Passage
1 would
mostBeneath
likely question?
the
French
Revolution.
the(B)
French
Revolution.
III only
ed A
the sentence are five words or sets of words labeled A
(C)
I
and
(A)
Public
funding
of
the
arts
increases
the
danger
of
(A) invoked
investigated
(C) conceived
when
(A) invokedII only
(B) investigated
(C) conceived
through E. Choose
the word or(B)
set of
words that, when
(D)
and III only
censorship.
(D) coveted
denounced
he
(D) II
coveted
(E) denounced
inserted in the sentence,
best fits (E)
the meaning
of the
(E) I, II, and III
(B)asArtistic
sentence
a whole.creation should not involve taking finanrisks.
4.cial
Luisa
worked with extreme precision, ------- that
4. Luisa worked with extreme precision, ------- that
Example:
(C) Artists
appreciate
support
served
her wellfinancial
in her law
career.more if they
served her well in her law career.
it the
afterdispute,
suffering
hardships.
sed
Hoping to earn
------negotiators
proposed
(A) a meticulousness
(B)have
an effrontery
(A) a meticulousness
(B) an effrontery
(D) Administrators
of arts
funding
both
a compromise
that an
they
felt
would
be
------tohigher
both artistic
(C)
inhibition
(D)
a
litigiousness
(C) an inhibition
(D) a litigiousness
standards
than
the
general
public.
labor and management.
(E) isanconducive
impetuousness
(E) an impetuousness
(E) Democracy
to artistic self-expression.
(A) enforce . . useful
In 1916 Yellowstone National Park had only 25 bison, 5. In 1916 Yellowstone National Park had only 25 bison,
(B) end 5.
. . divisive
but .the
population has since ------- to more than 2,000.
but the population has since ------- to more than 2,000.
(C) overcome
. unattractive
(D) extend (A)
. . satisfactory
dispersed
(B) mediated
(C) attenuated
(A) dispersed
(B) mediated
(C) attenuated
(E) resolve . . (D)
acceptable
burgeoned
(E) reconciled
(D) burgeoned
(E) reconciled
iences,
ative
6. Though surgeon and researcher Charles Drew never 6. Though surgeon and researcher Charles Drew never
1. Originally ------- mainly by young, urban audiences,
enjoyed celebrity, he truly deserves to be ------- for his
enjoyed celebrity, he truly deserves to be ------- for his
rap music was ultimately ------- by its appreciative
life’s achievements.
life’s achievements.
listeners of all ages across the country.
(A) mollified
(B) lionized
(C) accosted
(A) mollified
(B) lionized
(C) accosted
(A) admired . . embraced
(D) galvanized
(E) vilified
(D)
galvanized
(E)
vilified
(B) performed . . condemned
(C) derided . . ignored
(D) appropriated . . relinquished
(E) applauded . . instigated
author of 2. It was out of ------- that Professor Green, the author of
scribed
several highly respected books in his field, described
himself to his colleagues as -------.
If you finish before time is called, you may check your work on this section only.
(A) embarrassment . . a paragon Do not turn to any other section in the test.
(B) magnanimity . . an avenger
(C) insolence . . a pedant
(D) egotism . . an apprentice
(E) modesty . . a dilettante
STOP
- 38 -
NO TEST MATERIAL ON THIS PAGE.
- 39 -
greatest
o that
?
SECTION
9 A garden has r parallel rows of plants, with 5 plants in
13. Aare
garden
hasonr aparallel
rows of
plants,
with
5 plants in13.
11. Six points
placed
circle. What
is the
greatest
Timehow
— 20
minutes
x plants
arebe
added
to so
each
row,
many
each
row. Iflines
number of
different
that can
drawn
that
each row. If x plants are added to each row, how many
r and
x?
then two
be inofthe
garden,
in terms of16
Questions
plants will then be in the garden, in terms of r and x ?
each lineplants
passeswill
through
these
points?
(A) 5rx
(A) 12 (A) 5rx
to Section
5r x 9 (page 7 of your answer sheet to answer
x questions in this section.
(B) 5r the
(B)Turn
15 (B)
(C) 5r rx
(C) 25 (C) 5r rx
r
x
(D)
5
5
x
(D)
5r the5choices
(D)
30
Directions: For this section, solve each problem and decide which is the best of
given. Fill in the corresponding
5 You
x may use any available space for scratchwork.
(E) rsheet.
(E) r 5 x
36 answer
circle(E)
on the
14. Three lines are drawn in a plane so that there are
14. Three lines are drawn in a plane so that there are
exactly three different intersection points. Into how
exactly three different intersection points. Into how
many nonoverlapping regions do these lines divide the
many nonoverlapping regions do these lines divide the
plane?
plane?
(A) Three
(B) Four
1. A community
college charges an activity fee of $4.00
(C)andFive
per student
has a student body of 8,200 students.
(D)
dinate on 12. If
Point
P student
is
theSix
point
the what
greatest
y-coordinate
on
every
payswith
the fee,
is the
total amount
(E)
oordinate
theactivity
semicircle
shown
above.
is the x-coordinate
in
feesSeven
collected
fromWhat
the students?
of point Q ?
(A)
$32.80
(A) −3.5$328.00
(B)
(B) −3$3,280.00
(C)
2.5
(C) −$32,800.00
(D)
−2
(D) $328,000.00
(E)
(E) −1.5
(A)
(B)
(C)
(D)
(E)
Three
Four
Five
Six
Seven
2. In the figure above, OA OC and OB OD.
If x = 35, what is the value of z ?
(A)
(B)
(C)
(D)
(E)
- 40 -
55
45
35
30
25
is the greatest
awn so that
oints?
COMMON
4.
13. Aare
garden
hasonr aNAIL
parallel
rows of
plants,
with 5 plants in13.
11. Six points
placed
circle.SIZES
What
is the
greatest
x plants
arebe
added
to so
each
row, how many
each
row. Iflines
number of
different
that
can
drawn
that
Length
then in
beInches
inofthe
garden,
in terms of r and x ?
each lineplants
passeswill
through
two
these
points?
Size
(A)
(B)
(C)
(D)
(E)
12
15
25
30
36
(A)
(B)
(C)
(D)
(E)
5rx
5r
5r
5r
r
2d
1
x
1
1
3drx
4
5x
1
4d5 x 1
2
1
2
8d
2
10d
3
In garden
a sequence
numbers,
theof
first
number
and in
A
has rofparallel
rows
plants,
withis52plants
each
number
after
the
first
is
2
more
than
3
times
many
each row. If x plants are added to each row, how the
preceding
What
is the fourth
number
in the
x?
plants
will number.
then be in
the garden,
in terms
of r and
sequence?
(A) 5rx
(A) 5r14 x
(B)
(B) 5r17 rx
(C)
(C) 5r38 5x
(D)
(D) r80 5 x
(E)
(E) 242
3. Some common nail sizes and their corresponding
lengths are shown in the table above. If nail sizes
from 2d up to 10d increase by a constant length for
each increase of 1d in size, what would be the length,
in inches, of a 6d nail?
(A) 2
1
4
(B) 2
1
8
(C) 2
14. Three lines are drawn in a plane so that there are
14. Three lines are drawn in a plane so that there are
7
(D) 1 exactly three different intersection points. Into how
exactly three different intersection points. Into how
8 many nonoverlapping regions do these lines divide the
many nonoverlapping regions do these lines divide the
5. plane?
If x = 3 y and y = 4z, what is x in terms of z ?
3 plane?
(E) 1
4 (A) Three
3
(A)
(A) Three
z
(B) Four
(B) Four
4
(C) Five
(C)
(B) Five
z
(D)
(D) Six
-coordinate on 12. Point P is
theSix
point with the greatest y-coordinate on
(E) Seven
4
(E)
e x-coordinate
the semicircle
shown above. What is the x-coordinate
(C) Seven
z
3
of point Q ?
(A)
(B)
(C)
(D)
(E)
(D)
−3.5
−3
−2.5
−2
−1.5
7z
(E) 12 z
- 41 -
greatest
o that
?
13.
Aare
garden
hasonr aparallel
rows
of
plants,
withthe
5 plants in13.
11.
Sixthepoints
placed
circle. of
What
is the
7.
6. If
average
(arithmetic
mean)
5 and
r isgreatest
7 and
plants
are
added
toof
each
row,s how
row.
Iflines
number ofeach
thatiscan
drawn
so
average
3different
and
s is
3,x what
thebe
average
rthat
and
? many
then two
be inofthe
garden,
in terms of r and x ?
each lineplants
passeswill
through
these
points?
(A) 3
(A) 12
(B)
5 (A) 5rx
(B) 15
(C)
6 (B) 5r x
(C)
25
(D) 9 (C) 5r rx
(D) 12
30 (D) 5r 5x
(E)
(E) 36 (E) r 5 x
m
2 m
Am
garden
If
> 0, has
thenr mparallel−rows of
= plants, with 5 plants in
m
m
to each row, how many
each row. If x plants are added
plants0 will then be in the garden, in terms of r and x ?
(A)
(A)
(B)
(B)
(C)
(C)
(D)
(D)
(E)
15rx
5r
m
5r2
m
5r
2
mr
x
rx
− 51x
5 x
14. Three lines are drawn in a plane so that there are
14. Three lines are drawn in a plane so that there are
exactly three different intersection points. Into how
exactly three different intersection points. Into how
many nonoverlapping regions do these lines divide the
many nonoverlapping regions do these lines divide the
plane?
plane?
(A) Three
(B) Four
(C) Five
(D)
dinate on 12. Point P is
theSix
point with the greatest y-coordinate on
(E)
oordinate
the semicircle Seven
shown above. What is the x-coordinate
of point Q ?
(A)
(B)
(C)
(D)
(E)
(A)
(B)
(C)
(D)
(E)
−3.5
−3
−2.5
−2
−1.5
- 42 -
Three
Four
Five
Six
Seven
is the greatest
awn so that
oints?
Questions
to has
theonfollowing
figures
and
13.8-9Arefer
garden
r aparallel
rows
of
plants,
with 5 plants in13. A gardenLENGTH
11. Six points
are
placed
circle. What
is the
greatest
has r parallel
of plants,
with 5 plants in
OF Arows
YOUNG
SNAKE
information.
x plants
arebe
added
to so
each
row, how many
each
row. Iflines
number of
different
that can
drawn
that
each row. If x plants are added to each row, how many
then two
be inofthe
garden,
in terms of r and x ?
plants
each lineplants
passeswill
through
these
points?
Agewill then be2in the garden,
3
4in terms5 of r and
6 x?
(in months)
(A) 5rx
(A) 12 (A) 5rx
(B)Length
5r x
(B) 15 (B) 5r x
4
9
11
12
12.5
5r rx
(C) 25 (C) 5r rx
(in(C)
centimeters)
(D) 5r 5x
(D) 30 (D) 5r 5x
(E) r 5 x
(E) 36 (E) r 5 x
10. Which of the following graphs best represents the
information in the table above?
(A)
(B)
The figure on the left is called an ell. The lengths of
some of its sides are given, and all the angles are right
angles. For any positive integer n, an n-ell is the figure
formed by positioning n ells adjacent to each other as
shown in the 3-ell on the right.
8. What is the perimeter of the 3-ell?
(A)
(B)
(C)
(D)
(E)
18
21
(C)
(D)
24
27
30
14. Three lines are drawn in a plane so that there are
14. Three lines are drawn in a plane so that there are
exactly three different intersection points. Into how
exactly three different intersection points. Into how
many nonoverlapping regions do these lines divide the
many nonoverlapping regions do these lines divide the
plane?
plane?
(A) Three
(B) Four
(C) Five
(D)
-coordinate on 12. Point P is
theSix
point with the greatest y-coordinate on
(E)
e x-coordinate
the semicircle Seven
shown above. What is the x-coordinate
of point Q ?
(A)
(B)
(C)
(D)
(E)
(E)
(A) perimeter
−3.5
9. The
of an 80-ell is 326 and the perimeter of a
3 86. What is the perimeter of a 100-ell?
(B) −is
20-ell
(C) −2.5
(A)
−2
(D) 406
(B)
−1.5
(E) 409
(C) 412
(D) 416
(E) 430
- 43 -
Three
Four
Five
Six
Seven
greatest
so that
?
Add
to
2x.
a one-cent
sale, a rows
shopper
pays the
regular
13.3yAare
garden
hasonr aparallel
rows of
plants,
with 5 plants in13.
11. 1.
Six
points
placed
circle. What
is the
greatest
13. During
A garden
has r parallel
of plants,
with
5 plants in
2.
Multiply
therow.
sum Ifby
of vegetable
oilto
and
pays
$0.01
x2.plants
arebe
added
to so
each
row, how many price
each
are added
each
row,
howfor
many
number
of
different
lines
that can
drawn
that
each for
row.a bottle
If x plants
3.
Subtract
x − will
2through
y from
the
product.
aplants
second
bottle.
priceinofterms
the vegetable
then two
be in
garden,
in terms of r and x ?
will
then Ifbethe
in regular
the garden,
of r and x ?
each
lineplants
passes
ofthe
these
points?
oil is $1.89, how much per bottle does the shopper
(A)
5rx are followed in order, which of the
(A) by
5rxbuying two bottles at this sale?
save
11. If(A)
the 12
steps
above
r
x
(B)
5
r x
(B)
5
(B)
15
following is a simplified expression for the result?
(A)
(C) $0.01
5r rx
(C) 25 (C) 5r rx
(B)
(A)
4 y (D)
+ 3 x5r 5x
(D) $0.94
5r 5x
(D) −30
(C)
r
5
x
(E)
r 5 x
(E) $0.95
(E) 36 y + 3 x
(B)
(D) $0.96
(C) 4 y + 5 x
(E) $1.89
(D) 5 y + 3 x
(E) 8 y + 3 x
14. Three lines are drawn in a plane so that there are
14. Three lines are drawn in a plane so that there are
r + t three5 different intersection points.
r
exactly three different intersection points. Into how 14. Ifexactly
= , what is the value of ? Into how
many nonoverlapping regions do these lines divide the
r − nonoverlapping
t
t lines divide the
2
many
regions do these
plane?
plane? 7
(A) −
(A) Three
3
(A) Three
12. If k is a positive
integer, which of the following is
(B) Four
(B) Four
k
k
+ 3 ?
equivalent(C)
to 3Five
−1
(B)
(C) Five
(D)
Six
k
(D)
Six
dinate on 12. (A)
Point2 !P3 is the point with the greatest y-coordinate on
3
(E) Seven
(C)
(E) Seven
oordinate
the semicircle
shown above. What is the x-coordinate
2k
7
(B)
3 Q?
of point
k
(D)
1
(C)
(A) 6−3.5
(B) 6−23k
7
(D)
(E)
(C) −22k.5
3
(E)
(D) 9−2
(E) −1.5
- 44 -
is the greatest
awn so that
oints?
13.circular
Aare
garden
hasonr ahas
parallel
rows
of
plants,
with 5 plants in16.
11. A
Sixright
points
placed
circle.
What
is the
greatest
13.
cylinder
a base
of circumference
15.
x
plants
are
added
to
each
each
row.
If
number
of
different
lines
that
can
be
drawn
so
that
the how many
8 . If the volume of the cylinder is 128 , what isrow,
then two
be inofthe
garden,
in terms of r and x ?
each lineplants
passeswill
through
these
points?
height?
(A)
(B)
(C)
(D)
(E)
12
4
15
8
25
12
30
16
36
32
(A)
(B)
(C)
(D)
(E)
5rx
5r x
5r rx
5r 5x
r 5 x
A garden
has r parallel
rows
plants,
In
the xy-coordinate
plane,
theofgraph
of with
y 5 plants
x 2 9in
each row. If x plants are added to each row, how many
intersects line at p, 5 and t , 7 . What is the
plants will then be in the garden, in terms of r and x ?
least possible value of the slope of ?
(A) 5rx
(A)
(B) 5r 6 x
(C) 5r 2 rx
(B)
(D) 5r 5x
(C)
2
(E) r 5 x
(D)
6
(E) 10
14. Three lines are drawn in a plane so that there are
14. Three lines are drawn in a plane so that there are
exactly three different intersection points. Into how
exactly three different intersection points. Into how
many nonoverlapping regions do these lines divide the
many nonoverlapping regions do these lines divide the
plane?
plane?
(A) Three
(B) Four
(C) Five
(D)
y-coordinate on 12. Point P is
theSix
point with the greatest y-coordinate on
(E)
e x-coordinate
the semicircle Seven
shown above. What is the x-coordinate
of point Q ?
(A)
(B)
(C)
(D)
(E)
(A)
(B)
(C)
(D)
(E)
Three
Four
Five
Six
Seven
−3.5
−3
−2.5
−2
−1.5
STOP
If you finish before time is called, you may check your work on this section only.
Do not turn to any other section in the test.
- 45 -
NO TEST MATERIAL ON THIS PAGE.
- 46 -
sumptionironment at a
as Myanmar.
SECTION
10My grandfather never learned to use a calculator, as he
9. living
My grandfather
never learned
to use a calculator,
as he9.
5. Someone
in a technological,
consumption— 10
shopsprobably
he can accurately
his grocery
bill
in minutes
his shops he can accurately compute his grocery bill in his
oriented culture
taxes the compute
environment
atTime
a
head to
within
a dollar.such as Myanmar. 14 Questions head to within a dollar.
rate many times
that
of a country
r
as he shops
(A) that of(A)
a country
such as Myanmar
(A) as he shops
to
Section
10 (page
7) oflike
your
answer sheet to(B)
answer
the questions in this section.
y like Myanmar (B)Turn
(B)
while living
shopping
that of
someone
in a country
Myanmar
while shopping
tance
(C) find
but as
he shops for instance
(C) what you
in Myanmar,
(C) but as he shops
For
each
question
in this
section, select the best answer from(D)
among
the choices
and fill in the corresponding
Directions:
mar
(D)in
therefore,
when
(D) the rate
a country
such
asshopping
Myanmar
therefore,
when given
shopping
e
(E) ofhowever,
he for
shops
so that
circle
answer
sheet.
(E)on athecitizen
Myanmar,
instance
(E) however, he shops so that
2.
10.
The
first
African
American
woman
win the Pulitzer10.
6. Airport
runways
must
be correctness
constantly
swept
cleartoof
The
following
sentences
test
and
effectiveness
Prize
for
poetry,
Gwendolyn
Brooks
with
her
special
trash
and
other
debris
that
could
be
sucked
into
a
jetof expression. Part of each sentence or the entire sentence
interest
encouraging
young
poets.
engine intake
or it in
could
cause a serious
is underlined;
beneath
each
sentence
are
fiveaccident.
ways of
phrasing
the
underlined
material.
Choice
A
repeats
the
ccident
her
special
interest
in encouraging
(A) intake(A)
or itwith
could
cause
a serious
accident
original
phrasing;
the
other
four
choices
are
different.
If
ent
a special
interest
in encouraging
(B) intake,(B)
thishad
causes
a serious
accident
you
think
the
original
phrasing
produces
a
better
sentence
t
a special
interest, which was to
(C) intake(C)
and having
cause ahad
serious
accident
than(D)
any intake,
of the alternatives,
choice
A; if not, select
nt
encourage
preventing
aselect
serious
accident
one (E)
of theintakes
other
choices.
ident
(D)
had a aspecial
in encouraging
andwho
avoiding
seriousinterest
accident
(E)
she
had
a
special
interest
to
encourage
In making your selection, follow the requirements
of
3.
would enhance standard
7. In believing
that
firsthand
experience
would
written English; that is, pay attention to enhance
grammar,
mbus, Professorchoice
11.
Although
fascinated
by
chance
and
coincidence,
Paul
the of
credibility
of
his
biography
of
Columbus,
Professor
11.
words, sentence construction, and punctuation.
s’ first voyage. Your
Auster’s
novels
written
withfirst
careful
attention to
Morison
retraced
route
of
voyage.
selection
shouldthe
result
in are
theColumbus’
most effective
style
and
balance.
sentence—clear
and
precise,
without
awkwardness
or
(A) In believing that
ambiguity.
(A) that
Paul Auster’s novels are written
(B) Believing
Paul
(C) In his(B)
belief
thatAuster’s novels were written
EXAMPLE:
(C) Paul
(D) He believed
thatAuster writes his novels
(D)
Paul
Auster is aher
writer
(E)
By
believing
that
Laura Ingalls Wilder
published
first book
(E)
Paul
Auster
had
wrote
and she was sixty-five years old then.
is more often an8. Except in mathematics, absolute proof is more often an
sheEarly
was American
sixty-five
years
oldreached,
thennot so
factories
did
much
ched, a fact that (A)
12.
idealand
to12.
be
sought
than a goal
to be
a fact
thatreplace
(B)
when
she
was
sixty-five
standards of
household
manufacturing
butstandards
complement
the courts recognize by setting varying
of it.
(C)
at for
agedifferent
sixty-five
years
proof
kinds
ofold
cases.
(A)reaching
but complement
(D) upon the
of sixty-five years
4.
setting
(B)
as complement
(A) ata the
facttime
that when
the
courts
recognize
by setting
(E)
she was
sixty-five
but they
complemented
(B) which(C)
the courts
recognize
and set
urts are setting
(D)is and
they complemented
(C) and this
recognized
when the courts are setting
when they set
as they were
to complement
(D) and it(E)
is recognized
by the
courts when they set
1. (E)
In their
to make
beachfront
widely
etting
andzeal
the courts
recognize
thisliving
fact setting
available, developers have overbuilt, thereby they
endanger fragile coastlines.
ept clear of
ked into a jets accident.
(A) overbuilt, thereby they endanger fragile coastlines
(B) overbuilt they endanger fragile coastlines as a
result
(C) overbuilt and thereby have endangered fragile
coastlines
(D) overbuilt; fragile coastlines endangered thereby
(E) overbuilt, the fragile coastlines are endangered
by this
Hawaii’s
Haleakala,
being more
than
high,
The
first African
American
woman
to 10,000
win thefeet
Pulitzer
and the
dormant
volcano.
Prize
forworld’s
poetry, largest
Gwendolyn
Brooks
with her special
interest
in encouraging
youngthan
poets.
(A) Haleakala,
being more
10,000 feet high and
(B)
Haleakala,
more
than
10,000
feet high, it is
(A) with her special interest in encouraging
(C)
Haleakala
which
is
more
than
10,000 feet high,
(B) had a special interest in encouraging
being
(C) having had a special interest, which was to
(D) Haleakala,
encouragemore than 10,000 feet high, is
(E) who
Haleakala,
more than
10,000
feet high; it is
(D)
had a special
interest
in encouraging
(E) she had a special interest to encourage
I do not blame Leslie for her anger yesterday, being it
was her plan
and sheby
should
have
for it. Paul
Although
fascinated
chance
andcredit
coincidence,
Auster’s
novels are
written
careful
to
(A) yesterday,
being
it waswith
her plan
andattention
she should
style and
balance.
have
credit for it
(B) Paul
yesterday,
being
that are
shewritten
should have credit for it
(A)
Auster’s
novels
when
it
was
her
plan
(B) Paul Auster’s novels were written
(C) Paul
yesterday
it was
her plan, for which they
(C)
Austerwhen
writes
his novels
should
giveis her
credit for it
(D) Paul
Auster
a writer
(D) Paul
yesterday;
she should receive credit, it being
(E)
Austersince
had wrote
her plan
(E) yesterday:
was herdid
plan,
she should
have
Early
American itfactories
notand
so much
replace
received
credit
for
it
household manufacturing but complement it.
(A)
buttime
complement
At the
at which temperatures approach absolute
(B)
as
complement
zero, or -459.7° F, metals become highly conductive,
(C)
but they
complemented
and their
volume
shrinks dramatically.
(D) and they complemented
(A) as
Atthey
the time
temperatures approach
(E)
wereattowhich
complement
(B) When temperatures approach
(C) Since temperatures approached
(D) At the point temperatures had approached
(E) While temperatures approaching
5. Participants in the executive leadership workshop
expect a program of outstanding speakers and gaining
information about new approaches to management.
(A)
(B)
(C)
(D)
(E)
- 47 -
and gaining information
as well as information
as well as being informed
and also being informed
in addition, they expect to gain information
6. Someone
One of 9.
theliving
unforeseen
consequences
of the
editor’s
My
grandfather
never learned
to use
a calculator, as he10.
5.
in a technological,
consumption9.
management
style
is
that
it
leaves
so
little
room
for
shopsprobably
he can accurately
his grocery
bill in his
oriented culture
taxes the compute
environment
at a
innovation.
head to
within
a dollar.such as Myanmar.
rate
many times
that
of a country
(A)
that
it
leaves
so
little
for innovation
as he shops
(A) that of(A)
a country
suchroom
as Myanmar
(B)
that
they
leave
so
little
room
for innovation
Myanmar (B) that of(B)
while living
shopping
someone
in a country
like Myanmar
(C)
that
sufficient
room
is
not
left
their innovation
(C) find
but as
he shops forfor
(C) what you
in Myanmar,
instance
(D) the
thatrate
innovation
has so
little
room
left from it
(D)in therefore,
when
(D)
a country
such
asshopping
Myanmar
(E) ato citizen
leave
little
room
innovation
(E)soofhowever,
hefor
shops
so that
(E)
Myanmar,
for
instance
tionent at a
yanmar.
7.
ear of
6.
nto a jetdent.
nt
8.
enhance 7.
, Professor
t voyage.
re often an8.
a fact that
ards of
g
re setting
they set
g
9.
After
1907, residents
the Omaha
My
grandfather
neverof
learned
to useReservation
a calculator,could
as he
use
the
hospital
in
Walthill,
Nebraska,
it was bill in his
shops he can accurately compute his grocery
established
by aDr.
Susan LaFlesche Picotte, an Omaha
head
to within
dollar.
Indian.
(A) as he shops
(A) while
hospital
in Walthill, Nebraska, it was established
(B)
shopping
by
Dr.
Susan
(C) but as he shopsLaFlesche Picotte, an Omaha
Indian when shopping
(D) therefore,
(B) however,
hospital; ithewas
in Walthill,
(E)
shops
so that Nebraska and
established by Dr. Susan LaFlesche Picotte, an
For decades,
American
music
has inspired
Omaha
Indian
10.
TheAfrican
first
African
American
woman
win the Pulitzer10. The first
Airport
runways
must
be constantly
swept
cleartoof
African
American woman to win the Pulitzer
musicians
throughout
the
world,
including
in
Russia.
(C) hospital
thatGwendolyn
has been established
by her
Dr. special
Susan
Prizedebris
for poetry,
Gwendolyn
Brooks
with
trash and other
that could
be sucked
into
a jet-her special
Prize
for poetry,
Brooks with
LaFlesche
Picotte,
an
Omaha
Indian,
in
Walthill,
interest
in
encouraging
young
poets.
engine
intake
or
it
could
cause
a
serious
accident.
interest
in
encouraging
young
poets.
(A) including in Russia
Nebraska
(B) intake
including
ofcause
Russia
(A)
with
her
special
interest
in encouraging
(A)
or itthose
could
a serious
accident
(A)
her special
interest
in encouraging
(D) with
Walthill,
Nebraska,
hospital
where an Omaha
(C)
this
includes
Russia
a special
interest
in encouraging
(B) intake,(B)
thishad
causes
a serious
accident
(B) had a special interest in encouraging
Indian,
Dr.
Susan
LaFlesche
Picotte, established
(D) intake
one of(C)
which
is Russia
having
a special
interest, which was to
(C)
and
cause
ahad
serious
accident
(C) having had a special interest, which was to
it
(E) intake,
one example
beingaRussia
encourage
(D)
preventing
serious accident
encourage
(E) hospital established in Walthill, Nebraska, by Dr.
(D)
had a aspecial
in encouraging
(E) intakes
andwho
avoiding
seriousinterest
accident
(D) who had a special interest in encouraging
Susan LaFlesche Picotte, an Omaha Indian
By the end(E)
of the
eighteenth
century,
watchmaking
she
had a special
interest
to encourage
(E) she had a special interest to encourage
technology
had
greatly
improved,
and
they
were
In believing that firsthand experience would enhance
11. Although
Eating food
that has by
a high
concentration
of fat causes
standard
militaryby
11.equipment
Although
fascinated
chance andProfessor
coincidence, Paul 11.
the
credibility
of his for
biography
ofpersonnel.
Columbus,
fascinated
chance
and coincidence,
Paul
essentially
the
same
reaction
in
the
stomach
than if
Auster’s
novels
written with
careful
attention to
Morison
route are
ofequipment
Columbus’
voyage.
Auster’s novels are written with careful attention
to you
(A) and retraced
they
werethe
standard
forfirst
military
eat too
fast.
style
and
balance.
style
and
balance.
(A) Inpersonnel
believing that
(A) Paul
than if
you eatnovels are written
(B) Believing
so it was
equipment
forare
military
(A) standard
Paul Auster’s
novels
written
(B)
that
(A)
Auster’s
(B)
than
to
eat novels were written
personnel
to
have
watches
(B)
Paul
Auster’s
novels
were
written
(C) In his belief that
(B) Paul Auster’s
(C)
as
if
one
eatswrites his novels
(C) He
withbelieved
watches
included
in the standard
equipment
(C) Paul
Auster writes
his novels
(D)
that
(C) Paul Auster
(D)
as
eating
for
military
personnel
(D) Paul
Auster is a writer
(E) By believing
that
(D) Paul Auster is a writer
(E) Paul
as it does
when
(D) and watches
hadAuster
become
(E) Paul
hadstandard
wrote equipment for
(E)
Auster
had eating
wrote
military
personnel
Except in mathematics, absolute proof is more often an
12. Early
Not one
of the students
thenot
advanced
chemistry
(E) and
military
standard
Early
American
factories
did
not so
much
12.
American
factoriesindid
so much
replace class
ideal
to12.
befor
sought
thanpersonnel
a goal
toitbewas
reached,
a fact
thatreplace
have
passed
a
single
test
with
a
grade
better
equipment
it.
household
manufacturing
but
complement
the courts recognize by setting varying standards of
household manufacturing but complement it.than a C,
but the second half of the course will be easier.
proof for different kinds of cases.
(A)
but
complement
(A)
but complement
Nancy and Carlos will represent Central High in
(A) as
have
passed a single test with a grade better than a
as complement
(A)
a fact (B)
that competition,
the
courts recognize
by in
setting
(B)
complement
the swimming
their work
this having
C they complemented
(C)
but
they
complemented
(B)
which
the
courts
recognize
and
set
(C)
but
been excellent this year.
(B) and
havethey
managed
to pass a single test with better than
(D)is and
they complemented
(C) and this
recognized
when the courts are setting
(D)
complemented
C grade
(A) and
competition,
their were
work
in complement
this having
as they
to
(D)
it(E)
is recognized
by the
courts
whenbeen
they set
(E) asathey
were to complement
excellent
thisrecognize
year
(C) have passed a single test any better than
(E) and
the courts
this fact setting
(B) competition, they have done excellent work
a grade of C
this year in this
(D) has passed having better than a C grade on
(C) competition, for this year they have done
a single test
excellent work in this
(E) has passed a single test with better than
(D) competition, for their swimming has been
a C grade
excellent this year
(E) competition, their work as swimmers having
been excellent this year
- 48 -
13.
In neighborhoods
the learned
United
States,
sumption9. living
My grandfather
never
to use aone
calculator, as he14.
5. Someone
inthroughout
a technological,
consumption9.
can encounter
hundreds
oftaxes
different
rope-jumping
ironment at a
shops
he can accurately
his grocery
bill in his
oriented
culture
probably
the compute
environment
at a
games,
each
with
its
own
as Myanmar.
head
to
within
arules.
dollar.such as Myanmar.
rate
many
times
that
of
a country
(A) that
eachofwith
its
own
rulesas Myanmar
r
(A)
as he
shops
(A)
a country
such
(B) that
eachofhaving
theirliving
own rules
y like Myanmar (B)
(B)
while
shopping
someone
in a country like Myanmar
(C) what
whenyou
theyfind
each
their own
rules
stance
(C)
but
as
he shops
(C)
inhave
Myanmar,
for instance
(D)
which
has
its
own
rules
nmar
(D)
therefore,
when
shopping
(D) the rate in a country such as Myanmar
(E) athey
each
have
ruleshe
of for
their
own
e
(E) of
however,
shops
so that
(E)
citizen
Myanmar,
instance
A flurry
of do-it-yourself
books
market today
My
grandfather
never learned
to on
usethe
a calculator,
as he
are inspiring
to do their
own repairs.
shops
he can homeowners
accurately compute
his grocery
bill in his
head
to within
a dollar.
(A) are
inspiring
homeowners to do their own repairs
(B) as
areheinspiring
(A)
shops to homeowners about their own
repairs
(B) while
shopping
(C)
is
homeowners into doing their own
(C) butinspiring
as he shops
repairing
(D) therefore, when shopping
(D) however,
is inspiring
(E)
hehomeowners
shops so thatto do their own repairs
(E) inspiring homeowners to repair their own homes
wept clear of
The first
African
American
woman
win the Pulitzer10. The first African American woman to win the Pulitzer
6. Airport10.
runways
must
be constantly
swept
cleartoof
ked into a jetPrizedebris
for poetry,
Gwendolyn
Brooks
trash and other
that could
be sucked
into with
a jet-her special
Prize for poetry, Gwendolyn Brooks with her special
s accident.
interest
encouraging
youngaccident.
poets.
engine intake
or it in
could
cause a serious
interest in encouraging young poets.
accident
ent
nt
ent
cident
her
special
interest
in encouraging
intake(A)
or itwith
could
cause
a serious
accident
(A) with her special interest in encouraging
a special
interest
in encouraging
intake,(B)
thishad
causes
a serious
accident
(B) had a special interest in encouraging
a special
interest, which was to
intake(C)
and having
cause ahad
serious
accident
(C) having had a special interest, which was to
encourage
intake, preventing
a serious accident
encourage
(D)
had a aspecial
in encouraging
intakes
andwho
avoiding
seriousinterest
accident
(D) who had a special interest in encouraging
(E) she had a special interest to encourage
(E) she had a special interest to encourage
would enhance 7. In believing that firsthand experience would enhance
umbus, Professor the credibility
11. Although
chance andProfessor
coincidence, Paul 11. Although fascinated by chance and coincidence, Paul
of his fascinated
biography by
of Columbus,
us’ first voyage.
Auster’s
written withfirst
careful
attention to
Morison retraced
thenovels
route are
of Columbus’
voyage.
Auster’s novels are written with careful attention to
style and balance.
style and balance.
(A) In believing that
(A) that
Paul Auster’s novels are written
(B) Believing
(A) Paul Auster’s novels are written
Paul
(C) In his(B)
belief
thatAuster’s novels were written
(B) Paul Auster’s novels were written
(C) Paul
(D) He believed
thatAuster writes his novels
(C) Paul Auster writes his novels
(D) Paul
Auster is a writer
(E) By believing
that
(D) Paul Auster is a writer
(E) Paul Auster had wrote
(E) Paul Auster had wrote
is more often an8. Except in mathematics, absolute proof is more often an
factories
did not so
much
ched, a fact that
12. Early American factories did not so much replace
ideal to12.
be Early
soughtAmerican
than a goal
to be reached,
a fact
thatreplace
standards of
householdby
manufacturing
butstandards
complement
the courts recognize
setting varying
of it.
household manufacturing but complement it.
proof for different kinds of cases.
(A) but complement
(A) but complement
setting
as complement
(A) a fact (B)
that the
courts recognize by setting
(B) as complement
t
but they
complemented
(B) which(C)
the courts
recognize
and set
(C) but they complemented
ourts are setting
(D)is and
they complemented
(C) and this
recognized
when the courts are setting
(D) and they complemented
when they set
as they were
to complement
(D) and it(E)
is recognized
by the
courts when they set
(E) as they were to complement
setting
(E) and the courts recognize this fact setting
If you finish before time is called, you may check your work on this section only.
Do not turn to any other section in the test.
(A)
(B)
(C)
(D)
(E)
STOP
- 49 -
NO TEST MATERIAL ON THIS PAGE.
- 50 -
NO TEST MATERIAL ON THIS PAGE.
- 51 -